Сохранен 589
https://2ch.hk/sci/res/390896.html
24 декабря Архивач восстановлен после серьёзной аварии. К сожалению, значительная часть сохранённых изображений и видео была потеряна. Подробности случившегося. Мы призываем всех неравнодушных помочь нам с восстановлением утраченного контента!

Тупых вопросов о физике тред

 Аноним 06/10/16 Чтв 14:00:50 #1 №390896 
14757516504650.jpg
1. Если космонавт на МКС нагреет, скажем, паяльник и тут же выкинет его через условный мегабыстрый шлюз на мороз в вакуум, остынет ли этот паяльник? Ведь там какбе нет других тел, на которые он мог бы передать тепло? Или таки остынет за счёт того, что будет излучать в ИК диапазоне?

2. Парни, поясните за гравитационное взаимодействие. Мне этот вопрос спать не даёт. Вот допустим от ядра галактики Хуево-Кукуево до звезды по имени Цой тысяча световых лет. Однако гравитация от ядра галактики действует на звезду Цоя так сказать мнгновенно? Или нет?
Аноним 06/10/16 Чтв 15:41:45 #2 №390914 
>>390896 (OP)
>остынет ли этот паяльник
Нет, еще больше нагреется солнцем.
>будет излучать в ИК диапазоне
Да.

>Или нет
Или нет. Со скоростью света.
Аноним 06/10/16 Чтв 16:06:59 #3 №390919 
>>390914
короче единственный перенос тепла в космосе - это излучение в ИК, так?
Аноним 06/10/16 Чтв 16:27:45 #4 №390922 
>>390919
Не обьязательно ИК. Просто излучение.
Аноним 06/10/16 Чтв 16:41:57 #5 №390924 
>>390922
подтверждаю, например если какой нибудь железный лом сильно нагреть то будет красное видимое излучение
Аноним 06/10/16 Чтв 17:07:49 #6 №390928 
Что было бы, если бы скорость света была непостоянной?
Аноним 06/10/16 Чтв 19:26:12 #7 №390975 
>>390896 (OP)
Как намагнитить неодимовый магнит, размагниченный нагреванием его до точки Кюри?
Аноним 06/10/16 Чтв 22:07:23 #8 №391058 
>>390896 (OP)
Отличается ли чем то линейное сжатие от сжатия с ускорением?
Аноним 06/10/16 Чтв 23:11:11 #9 №391067 
Как ведет себя электромагнитное поле в окрестности бесконечно большой планеты, которая движется со скоростью света и при этом увеличивается в массе и соответственно в гравитации? Можно ли будет ловить радио?
Аноним 07/10/16 Птн 02:15:13 #10 №391093 
>>390896 (OP)
1. Надо считать. Действовать будет и нагрев от звезды, и охлаждение за счет излучения. Ставлю на еще больший разогрев.
2. Пока неизвестно. Гравитация либо мгновенна, либо равна скорости света в вакууме. Скорее всего равна с.
>>390928
Ничего, она и так непостоянна. В вакууме одна, в жидкости другая, в твердом теле другая.
>>390975
Остудить, чтобы домены снова выстроились в правильном порядке. Можешь дополнительно еще рядом поднести другой магнит для ускорения процесса.
>>391058
Не знаю.
>>391067
Если такая планета не превратится в черную дыру, то можно. А вообще тупой вопрос какой-то, математическую модель не построить, тут все что угодно может быть.
Аноним 07/10/16 Птн 09:42:09 #11 №391115 
>>390896 (OP)
1. А сам то ты как считаешь, когда тело излучает ИК лучи, оно остывает от этого?
А ещё вспомни про лампочку накаливания. Как считаешь, из-за чего она светит? Наверно из-за разогрева спирали до 2000 градусов. Так?
А почему лампочка гаснет почти мгновенно? Наверно потому что спираль так быстро остывает. А как спираль может так быстро остыть, если в лампочке вакуум, а теплопроводность спирали недостаточна для такого быстрого передачи тепла обратно в электропровода?

2. Гравитационное воздействие передаётся со скоростью света, как ви всё в этой вселенной. Даже небо, даже аллах.
Если например, дёрнуть ядро галактики, то колебание гравитацонного поля, произошедшее от этого дёрганья, дойдёт до Цоя и изменит траекторию его движения через тысячу лет.
Аноним 07/10/16 Птн 10:09:15 #12 №391118 
>>391115
а вон постом выше чел говорит, что, мол, неизвестно - может гравитационное взаимодействие и мнгновенно передавацца
Аноним 07/10/16 Птн 10:11:51 #13 №391121 
Не знаю где спрашивать, поэтому спрошу здесь. Если ты занимался когда-нибудь расчетом координат космических аппаратов, то доро пожаловать в тред: https://2ch.hk/spc/res/313385.html
Аноним 07/10/16 Птн 13:49:47 #14 №391152 
>>391118
Вариант со скоростью света самый логичный, потому что мгновенное распространение взаимодействия на всю вселенную - это уже как-то слишком. Поэтому, как я уже сказал, мор лайкли вариант - скорость света, однако подтверждения этой гипотезы еще не найдено.
Тот кун
Аноним 07/10/16 Птн 17:03:44 #15 №391226 
>>390896 (OP)
>остынет ли этот паяльник?
Немного, до 120 цельсиев примерно, дальше будет остывать в тени, а на свету опять нагреваться. Температура будет зависеть от альбедо, массы и площади поверхности паяльника.
>гравитация действует так сказать мгновенно?
Нет. Но скорость гравитации на много порядков больше скорости света. Например, от нас до галактики Андромеды она долетает за секунды.
Аноним 07/10/16 Птн 17:06:18 #16 №391230 
>>390928
Она и так не постоянна. Если бы она была постоянна, то вселенная бы сломалась.
Аноним 07/10/16 Птн 17:08:09 #17 №391233 
Что подразумевается под словом радиация? Гамма-излучение и только? И как вообще радиация действует на клетки? Ясен хер что хуево. Понятно что ионизирует но почему это плохо для органики? Что по факту делает излучение с клетками?
Аноним 07/10/16 Птн 17:30:36 #18 №391245 
>>391233
Радиация - это любое излучение.
>И как вообще радиация действует на клетки?
По-разному.
>Понятно что ионизирует но почему это плохо для органики?
Клетки бомбардируются частицами, как белый дом фугасными снарядами в 93-ем. После такого нормальную работу собрания депутатов не восстановить.
Аноним 07/10/16 Птн 18:22:05 #19 №391257 
>>391245
Можно хоть немного конкретики? Ты мне ничего нового не сказал. Я пришел сюда просвещаться а не слушать какие-то сравнения для дебилов. Что конкретно делает излучение с клеткой? Я вот за пару минут в интернете узнал что происходит радиолиз. Тоесть излучение вырывает электрон из молекулы и взаимодействует с другой и в итоге образуются свободные радикалы — соединения, которые также обладают высокой химической активностью.
Аноним 07/10/16 Птн 19:18:20 #20 №391269 
>>391257
Разрушает днк.
Аноним 07/10/16 Птн 21:53:56 #21 №391292 
>>391226
возможно ли в таком случае каким-либо образом использовать гравитационное взаимодействие для передачи информации (напрмиер для связи)? Я не говорю о конкретике, я теоритически.
Аноним 07/10/16 Птн 23:03:50 #22 №391307 
>>391292
Ну как ты себе представляешь генерацию переменной массы?
Аноним 07/10/16 Птн 23:06:25 #23 №391308 
>>391307
Налеплять и отлеплять кусок жвачки от переговорного устройства.
Аноним 08/10/16 Суб 00:59:38 #24 №391318 
>>390919
Не обязательно. Можно преобразовать тепловую в электрическую и обратно.
Аноним 08/10/16 Суб 01:03:50 #25 №391319 
>>391257
Излучение в зависимости от того какая частица летит может разных дел натворить, всё зависит от проникающей способности частицы. Так вот, если тебя вдоль насквозь прошибёт какой нибудь нейтрон, он может разворотить пару сотен тыщ клеток в разных местах, некоторые клетки повредятся ДНКой, помножив этот процесс на 10000 ты получишь что то типа вероятности появления раковой клетки где днк повредилась в определённых местах.
Аноним 08/10/16 Суб 09:55:38 #26 №391359 
>>391318
я имею в виду - для нагретого тела как такового, скажем, для звезды, в которой угасла термоядерная реакция.
А так-то да, на тепловом элементе все эти вояджеры и работают, насколько я пнимаю.
Аноним 08/10/16 Суб 10:46:41 #27 №391364 
>>391359
Если ты имеешь в виду спутники что с изотопами в качестве топлива, там вроде как радиацией энергию вырабатывают, а не тепловым излучением.
Аноним 08/10/16 Суб 21:09:55 #28 №391465 
>>391364
Чочочоч? Нет. Там активные изотопы греют тепловой элемент. Как ты из радиации получишь электричесво напрямую, лол
Аноним 08/10/16 Суб 21:24:48 #29 №391471 
Сможет ли существовать искусственно созданная планета формой отличная от сферы? Например кубическая или пирамдальная?
Аноним 09/10/16 Вск 10:41:56 #30 №391519 
>>391465
Я не помню, есть некая хуйня по аналогии солнечных батарей, там элементик из 1 грамма самого дорогого изотопа на планете, который по тому дорогой что живёт не долго и распадается фоня элементарными частицами аки маленькое солнышко.
Аноним 09/10/16 Вск 11:28:30 #31 №391523 
14760017105520.gif
>>391465
>Как ты из радиации получишь электричесво напрямую, лол
Дорого и хуёво.
Аноним 09/10/16 Вск 11:58:16 #32 №391525 
14760034963470.jpg
>>391308
Аноним 09/10/16 Вск 12:12:09 #33 №391531 
>>391523
Ну, пускай эти твои лепестки заряженные,
их растопыривает кулоновское отталкивание, и пускай они касаются фольги.
Тогда на заземлении плюс. А минус где?
Аноним 10/10/16 Пнд 15:29:39 #34 №391949 
>>391307
вон там две чд крутились да волны эти создавали гравитационные. как нибудь так. только все же сдается мне что ты пиздишь, и скорость гравитации не больше световой.
Аноним 10/10/16 Пнд 15:39:39 #35 №391950 
http://ziv.telescopes.ru/rubric/hypothesis/?pub=1 вот тут про скорость гравитации. которая не быстрее светы. правда многа буков
Аноним 10/10/16 Пнд 16:15:43 #36 №391957 
>>391471
да, почему нет? маза в том что случайно образовывающемуся говну, из мельчайших частиц, энергетически более выгодно собираться сферически. как капле воды. но если ты создашь квадратную планету из железа, будет квадратной, хуле ей?
Аноним 10/10/16 Пнд 16:20:05 #37 №391959 
>>391957
> из железа
А она не коллапсирует?
Аноним 10/10/16 Пнд 16:27:28 #38 №391963 
>>391959
ну из железобетона тогда лол. или чего угодно другого. не обязательно опять же полностью из железа, можно с пустатми, чтоб масса была по меньше. да кстате подозреваю, что даже монолитный кусок железа обьемом с землю не коллапсирует. вообще говоря странный вопрос. есть же какая нибудь сраная и тупая форумула, типа коллапс будет если плотность у дерьма такая то, обьем этого дерьма такой то. и подозреваю что плотность там должна быть много больше чем у сраного железа. много много. но гуглейть конечна же лень, можешь прогуглить да подставить значения плотности железа да обьема земли.
Аноним 10/10/16 Пнд 16:46:08 #39 №391970 
14761071685010.png
Может ли такая штука снизить урон от прямых ударов тупым предметом? Держится сама как на рисунке сверху, работает по принципу пружины
Аноним 10/10/16 Пнд 16:53:58 #40 №391973 
>>391970
с точки зрения теоретической физики да. с точки зрения практической же только повысит. отпиздят за ношение хуя на голове лол. еще и выебут
Аноним 10/10/16 Пнд 16:55:18 #41 №391974 
>>391970
Ты изобрёл одностороннюю каску. Иди получай шнобелевку.
Аноним 10/10/16 Пнд 17:11:55 #42 №391984 
>>391465
> радиации получишь электричесво напрямую
радиация это ЭМ излучение, т.е возмущение ЭМ поля. и допустим есть разность потенциалов. разве это не электричество из радиции. антенна например.
Аноним 10/10/16 Пнд 17:13:14 #43 №391985 
>>391974
2D каску
Аноним 10/10/16 Пнд 19:46:53 #44 №392004 
>>391974
Если эту штуку прикрепить на каску то тогда вообще охуенчик же выйдет
Аноним 10/10/16 Пнд 19:58:04 #45 №392005 
>>392004
выйдет хуй на каске. за него опять же выебут. если уж так хочется сделать хуйню, то нацепи каску на каску
Аноним 10/10/16 Пнд 20:00:57 #46 №392006 
>>391984
а чего антенна? любой штырь будет тебе антенной. а радиация твоя фонит во все стороны. и даже если ты умудришься её направить только в одну.... погоди - ты что открыл новый способ добычи электроэнергии? тоесть все энергетики, что были до тебя не додумались об этом? ох щи, сказки уровня бэ, сосач на острие науки. сап саентач, есть одна радиация...
Аноним 11/10/16 Втр 14:36:35 #47 №392117 
14761857956500.png
>>392006
ты че еблан. ты как думаешь мы кпк заряжаем? не заряжать же эту хуйню через розетку каждый раз
Аноним 11/10/16 Втр 17:01:50 #48 №392147 
>>392006
какой новый способ. ты просто не понял. консервативность зарядов сохраняется. волны возбуждают поле в антенне, это ее электроны порождают электричество в приемнике. но таки да, радиация -> электричество.
Аноним 12/10/16 Срд 14:30:34 #49 №392258 
>>392147
тащем-то электричество это направленное движение заряженных частиц. так что радиация и без антенны сама по себе электричество. и молния электричество. только толк на практике от обоих по теореме эскобара можно посчитать
Аноним 12/10/16 Срд 19:18:02 #50 №392301 
Когда будут блутус устройства, которые пренебрежительно уступают проводным в скорости? Хочу мышь, клаву и наушники без лага в онлайн фпс.
Аноним 12/10/16 Срд 19:57:01 #51 №392311 
>>392301
>Когда будут блутус устройства, которые пренебрежительно уступают проводным в скорости?
Когда блютуз откажется от аппаратного шифрования сигнала (т.е. никогда, потому что слушать вопли твоего джаггернаута в дотане соседям нахуй не впёрлось).
Аноним 12/10/16 Срд 22:25:12 #52 №392342 
14763003124460.jpg
Поясните за фазы токов и напряжений в переменном токе. То есть как можно себе приблизительно представить это синусоидальное движение напряжения и тока, почему одно отстает от другого, как напряжение зависит от тока при таком движении. Седня сидел на паре по элтеху и что-то глубоко задумался, параллельно осознав, что в принципе не понимаю, как эта вся хуета там себе движется. Или скиньте сайт, на котором для дцп-аутистов объясняют. С меня нихуя
Аноним 13/10/16 Чтв 14:19:04 #53 №392404 
>>391471
Есть какой-то предел массы у каждого материала, после которого гравитация заставляет тело принять сферическую форму
Аноним 14/10/16 Птн 11:25:24 #54 №392508 
Что будет с нашей Вселенной спустя количество лет, равное числу Грэма?
В википедии есть статья по вопросу, но там масштабы откровенно маловаты.
Аноним 14/10/16 Птн 11:58:59 #55 №392513 
>>392508
БОЛЬШОЙ РАЗРЫВ
Аноним 14/10/16 Птн 12:35:51 #56 №392521 
Посоны, можно ли сказать, что фотон - это квант светового поля?

И ещё. Если вся разница между световыми и электромангнитным излучением - это длина волны, то получается что те же фотоны являются ещё и квантами эдектромагнитного поля?
Чот какт не укладывается в голове.
Аноним 14/10/16 Птн 13:03:05 #57 №392523 
Если предположить, что человек это элементарная частица, то какой спин у него будет?
Аноним 14/10/16 Птн 14:33:21 #58 №392532 
https://www.youtube.com/watch?v=tgvR9_zviKI
Зачем леветирующему двигателю опора сбоку?
Аноним 14/10/16 Птн 16:18:06 #59 №392546 
>>392513
Но это гипотеза только.
Как я понял, сейчас мы не знаем, что будет наверняка со Вселенной в далёком будущем.
Аноним 14/10/16 Птн 17:03:50 #60 №392547 
>>392342
отстает для реактивной нагрузки только, но ты туда не лезь - сложна. а через резистор синус и синус, бегут заряды туда потом сюда, никто не остает, не парься.
Аноним 14/10/16 Птн 19:56:46 #61 №392564 
14764642064870.png
Что будет если привязать луну к земле нервущимся тросом?
Аноним 14/10/16 Птн 20:03:13 #62 №392565 
>>392564
грунт, за который держится тросс оторвётся
Аноним 14/10/16 Птн 20:24:13 #63 №392568 
>>392565
А если петлю накинуть, землю разрежет пополам? А если сетку накинуть?
Аноним 15/10/16 Суб 00:36:59 #64 №392589 
>>390896 (OP)

>>390924
Ты идиот? Ты думаешь если лом нагрелся и светится красным, то он излучает чистый монохроматический красный свет ? Ты ошибаешься так как нагретое тело в первую очередь излучает ик излучение а потом по мере роста энергии хаотического движения атомов энергия гамма квантов возрастает и добавляется видимый свет.
Аноним 15/10/16 Суб 00:47:01 #65 №392590 
>>391093
Ты дурачек что ли ?? Как гравитация может быть мгновенна ?? Для кого эксперименты проводили в 21 веке ?? То есть ты хочешь сказать что информация может передаваться мгновенной из одного конца вселенной до другой ? Ошибаешься, у гравитации есть конечная скорость распространения и это неопровержимый факт. Даже по эксцентриситет траектории планет косвенно можно говорить об этом.
Аноним 15/10/16 Суб 00:53:14 #66 №392592 
>>391093
Ты дурачек что ли ?? Как может тело в на орбите земли нагреться ?? Мощность ик излучения от солнца на такой маленькой площади несоизмеримо мало по сравнение с собственной. Надо баланс мощностей составить. Что бы нагретое тело на орбите земли излучало столько же тепла сколько оно получает от солнца. Вот тогда температура останется постоянной, но она будет гораздо ниже нуля.
Аноним 15/10/16 Суб 23:20:45 #67 №392779 
>>392521
Разница между "световым" и электромагнитным излучением только в том, что волны с частотой "светового" диапазона запускают химические реакции в рецепторах глаза, а другие - нет. Так то эти волны ничем больше не отличаются фотон не "светится", если что
Аноним 15/10/16 Суб 23:25:10 #68 №392780 
Вот вам довольно бытовой вопрос. Но он связан с физикой и долго меня мучает.
Почему капюшон спасает от ветра? Вроде как разницы быть не должно, но ветер в лицо слабее дует.
Аноним 15/10/16 Суб 23:27:56 #69 №392781 
>>392532
Затем, что идеально отцентровать эти "левитирующие" подшипники невозможно и ротор просто откинет в одну из сторон.
Аноним 16/10/16 Вск 00:34:05 #70 №392786 
>>392779
ну ведь не токо же глаза
Аноним 16/10/16 Вск 00:35:37 #71 №392788 
>>392780
он просто перестаёт дуть тебе в ухо.
И от этого кажется, что стало теплее => ты с меньшим страданием способен вынести ветер прямо в щщи.
Аноним 16/10/16 Вск 08:36:09 #72 №392807 
Если железо/сталь облучить потоком нейтронов, станет ли оно радиоактивным? Если да, то на сколько долго оно будет радиоактивным? И существует ли способ превратить такое радиоактивное железо в обычное, стабильное состояние?
Аноним 16/10/16 Вск 10:06:06 #73 №392830 
>>392807
https://ru.wikipedia.org/wiki/Наведённая_радиоактивность

Ну если переработать такое железо, то можно получить чистое железо. А уже избавиться от изотопов железа до приемлемого уровня не знаю, можно ли.
Аноним 16/10/16 Вск 11:28:39 #74 №392858 
1 При повышении напряжения, электроны реально начинают двигаться быстрее?
2 Можно ли собрать цепь с одним источником ЭДС в которой будут наблюдаться разные по величине токи?
Аноним 16/10/16 Вск 11:42:11 #75 №392864 
>>392858
Сила тока — физическая величина, равная отношению количества заряда
ΔQ, прошедшего за некоторое время Δt
через поперечное сечение проводника,
к величине этого промежутка времени.

1. Да.
2. Величина тока зависит от сопротивления нагрузки и внутреннего сопротивления источника ЭДС.
Источник: закон Ома.
Аноним 16/10/16 Вск 11:46:52 #76 №392866 
>>392864
>1 При повышении напряжения, электроны реально начинают двигаться быстрее?
>>392858
>1. Да.
По сути, ЭДС - это напряженность электрического поля между разноимённо-заряженными контактами источника ЭДС.
На заряд в электрическом поле, помимо силы Кулона действует ещё и сила Лоренца, если заряд движущийся, коим и является электрон.
Аноним 16/10/16 Вск 13:47:50 #77 №392887 
>>392858
>1
Нет. Напряжение на "скорость" электрона не влияет. Это лишь разница в количестве свободных электронов между двумя проводниками.

При увеличении напряжения возрастет только число электронов, которые будут двигаться по проводнику (с той же скоростью), то есть ток.
Аноним 16/10/16 Вск 16:37:43 #78 №392957 
>>391531
>А минус где?
Веперде.
Аноним 16/10/16 Вск 16:38:21 #79 №392958 
>>392568
Располосует.
Аноним 17/10/16 Пнд 09:37:37 #80 №393059 
>>392887
Закидайте меня говном, если я не прав.
Но электроны же никуда не двигаются, они колеблются на своих орбитах, передавая энергию за счёт электромагнитного поля. ????? или я не прав???))))))
Аноним 17/10/16 Пнд 15:17:35 #81 №393087 
>>393059
Они двигаются, но оче медленно. Вроде как колебаниями передают волны энергии. Но фиг знает точно, как это работает, довольно мутная модель.
Аноним 17/10/16 Пнд 16:08:26 #82 №393098 
>>393059
Сила тока — физическая величина, равная отношению количества заряда
ΔQ, прошедшего за некоторое время Δt.
Носителем заряда в проводнике - является электрон.
Чем больше электронов прошло через поперечное сечение проводника - тем больший заряд они перенесли, и тем больший ток.
Электроны же двигаются под действием электрического поля, коим является ЭДС.
В случае переменного напряжения, они колеблются, да.
Аноним 17/10/16 Пнд 16:37:45 #83 №393109 
>>393098
Окей, я понял тебя.

Не помню где читал, но вроде у Детлафа том 2 электричество и магнетизм. Так вот, там было описан эксперимент каких то учёных, не помню и в нём говорилось, что они экспериментально выяснили скорость движения электронов, если я не ошибаюсь то она составила где то 10^6. Однако что мы наблюдаем, что потенциал на другом конце проводника устанавливается почти мгновенно (то есть 10^9,скорость света).
Аноним 17/10/16 Пнд 16:43:10 #84 №393111 
>>393059

Электроны двигаются под действием СИЛЫ Лоренца -та сила, которая показывает действие ЭМ поля на заряд. и там ещё магнитное поле вдобавок, а не только электрическое.
Аноним 17/10/16 Пнд 16:45:28 #85 №393112 
>>393098
И да, я тебя спрашиваю не про силу тока, а про скорость свободных зарядов.
Аноним 17/10/16 Пнд 16:48:35 #86 №393115 



Дрейфовая скорость электронов[править | править вики-текст]
Скорость (дрейфовая) направленного движения частиц в проводниках зависит от материала проводника, массы и заряда частиц, окружающей температуры, приложенной разности потенциалов и составляет величину, намного меньшую скорости света. За 1 секунду электроны в проводнике перемещаются за счет упорядоченного движения меньше чем на 0,1 мм[10] - в 20 раз медленнее скорости улитки. Несмотря на это, скорость распространения собственно электрического тока равна скорости света (скорости распространения фронта электромагнитной волны). То есть то место, где электроны изменяют скорость своего движения после изменения напряжения, перемещается со скоростью распространения электромагнитных колебаний.
Аноним 17/10/16 Пнд 17:43:39 #87 №393137 
>>393111
это же сила с которой магнитное поле действует на движущийся заряд.
Аноним 17/10/16 Пнд 18:18:29 #88 №393156 
>>393098
Количество заряда, прошедшего за единицу времени прямо пропорционально скоросити движения носителей этого заряда.
Независимо от того, постоянный ток или переменный.
Аноним 17/10/16 Пнд 18:34:14 #89 №393162 
Если и конденсаторы и индуктивности - реактивные элементы, которые умеют запасать энергию а потом отдавать, то почему кондеры намного пиздатее. Они гораздо популярнее же в схемотехнике. Электрическая, магнитная, потенциальная, кинетическая, и чо.
Аноним 17/10/16 Пнд 19:48:38 #90 №393175 
>>393162
Потому что в них меньше энергии проёбывается.
Аноним 17/10/16 Пнд 21:25:28 #91 №393191 
>>393175
Так энергия же в обоих не проебывается вообще, а возвращается, ну мрожет чуток резистивных потерь в проволочке, но только если ток большой.
Балансирующая поебота Аноним 17/10/16 Пнд 21:55:11 #92 №393195 
14767305114780.jpg
14767305115351.png
14767305115362.png
Соорудил тут пикрил и теперь нужно заставить её работать. Не хочу брать готовые реализации - хочу изобретать велик. Для этого запилил эмулятор, в котором можно быстро и просто проверить алгоритм. Короче сейчас есть очередная идея алгоритма и вопрос.
Алгоритм включает вычисление определённого интеграла от косинуса для того, чтобы узнать какой импульс нужно передать тележке, чтобы переместить её из положения А в положение Б. То есть, если нужно переместить из наклона от вертикали в 30 градусов, то я использую интеграл косинуса от 150 до 180. К тому же, нужно учитывать влияние гравитации которая будет мешать восстанавливать равновесие. Потому что когда мне понадобится наклонять поеботу, бессердечная гравитация внезапно будет мне помогать таки снова мешать. inb4: - Зачем тебе специально наклонять-то? - Это понадобится для управления движением.
Короче реквестирую способ вычисления величины влияния гравитации во время перемещения.
Аноним 18/10/16 Втр 09:32:06 #93 №393273 
>>393137
Электрическое разгоняет либо замедляет, а магнитное отклоняет.
F=QE+Q[VxB]
Аноним 18/10/16 Втр 09:36:50 #94 №393274 
>>393162
Вся схемотехника основа на логических нулях и единицах напряжений,а кондёры позволяют добиваться нужных напряжений, регулировать их и т.д.
Индуктивности дают скачки токи + магнитное поле, которые нах никому не нужны.
Аноним 18/10/16 Втр 09:39:30 #95 №393275 
>>393195
Самый простой способ это экспериментальный, либо если нет возможности, то промоделировать систему в программе, чтобы посмотреть, как влияет гравитация. Советую COMSOL.
Аноним 18/10/16 Втр 11:47:34 #96 №393294 
14767804548110.png
>>393275
Спасибо, анон. У меня был Algodoo. Получается, чтобы переместиться обратно из Б в А, при нормальной гравитации, нужна угловая скорость в 3,762 раза выше угл. скорости в момент нахождения в точке Б. Масса тела на коэффициент не влияет. А вот если гравитация =1g, то коэффициент будет 3,692.
Аноним 18/10/16 Втр 12:15:32 #97 №393297 
>>393294
А ты учитывал, что там будет неупругий удар, эта хуёвина отскочит и будут колебания + сопротивление есть штыревого соединения или на чём она там будет вертеться.
Аноним 18/10/16 Втр 12:45:09 #98 №393300 
>>393297
Ну в итоге-то вместо соединения будет колесо. И я должен буду устанавливать скорость его вращения такую, чтобы бот придерживался определённого угла наклона (0 - если нужно просто балансировать на месте, и где-то +-10, если нужно ехать).
А тип удара и отскок в симуляторе ни на что не влияют так как максимальный крен бота не превысит и 45 градусов. В противном случае для стабилизации не хватит ни мощности двигателей, не сцепления колёс и бот упадёт.
Аноним 18/10/16 Втр 14:10:50 #99 №393303 
>>393195
Для тех кому интересно, есть видео про подобный, но гораздо более навороченный девайс.
https://www.youtube.com/watch?v=Ep2lNMic_fk
https://www.youtube.com/watch?v=cyN-CRNrb3E
Аноним 18/10/16 Втр 14:22:34 #100 №393307 
>>393303
А вот такое должно получиться:
https://www.youtube.com/watch?v=poWP-RpPa3g
Аноним 18/10/16 Втр 17:26:17 #101 №393370 
>>393274
То есть U>I? А почему нельзя все на токах делать.
Аноним 18/10/16 Втр 19:13:08 #102 №393398 
>>393307
Это ты для фана делаешь, или эта ебань имеет какое-то практическое применение?
Аноним 18/10/16 Втр 19:24:20 #103 №393401 
>>393398
Да просто поиграться.
Аноним 19/10/16 Срд 00:53:45 #104 №393437 
>>391970
лол
Аноним 20/10/16 Чтв 21:40:54 #105 №393768 
Ответьте уже кто-нибудь четко, гравитация это сила взаимодействия тел, или искривления пространства-времени?
Аноним 20/10/16 Чтв 22:00:23 #106 №393777 
>>393768
Тела искривляют пространство-время.
Аноним 20/10/16 Чтв 22:32:59 #107 №393781 
>>393768
вот есть у тебя 2 кирпича в вакууме. на большом расстоянии друг от друга. нихуя не происходит. ты пододвинул один кирпич к другому на метр, нихуя не произошло. пододвинул еще на километр, тоже. но когда-нибудь наступит момент, что два кирпича таки возьмут и сами притянутся друг к другу. и вот тут вся хуйня в том, что можно сказать - ебать гравитация притянула два кирпича друг к другу. а можно сказать - оба кирпича провалились в воронки, которые получились в пространстве, из-за того, что массы этих кирпичей шатали пространство, как вакабу макака. и как бы разницы нет - что так обьясняй, что сяк - при одинаковых исходных данных получается одинаковый результат.другой вопрос, что у света например массы нет, в отличие от кирпича. а вблизи черных дыр и он меняет траекторию. хотя распространяется прямолинейно. походу там сама прямая изгибается. поэтому скорее пространство изгибается, чем кирпичи просто притягиваются. хотя количественно, еще раз повторюсь, получается одно и то же.
Аноним 23/10/16 Вск 13:42:07 #108 №394031 
Какой массой должно обладать тело, чтобы начать сжиматься в сферу?
Аноним 23/10/16 Вск 13:52:01 #109 №394033 
>>394031
Как атом.
Аноним 23/10/16 Вск 15:12:29 #110 №394048 
>>393768
Гравитация это взаимодействие тел через пространство. Тела шатают пространство, точнее не пространство, а то что в нем находится (эфир, вакуум, пятый элемент, материя), но об этом принято не говорить, а принято маняврировать математическими абстракциями, типа искривлений пространства. Далее пошатанное пространство (точнее эфир) давит на другие тела. Как именно шатается среда вакуума, что в ней происходит, предстоит еще разбираться.
Аноним 23/10/16 Вск 15:37:27 #111 №394053 
14772262473760.png
Анон, выручай, есть задача и собсна ее решение в решебнике.( на пикче слева от красной линии) Я из решебника взял чертеж вроде все сошлось, в моем представлении силы должны были так же стоять. Но принес преподавателю, он сказал что все хуйня, давай по новой и показал как должны стоять силы( на пикче справа от красной линии). Окей, по логике я даже вроде и согласен. Но у меня чисто логический вопрос какого тогда хуя верхний брусок должен соскальзывать влево, если его сила трения тянет в туже сторону куда и искомая сила тянет нижний брусок? Вообще анон, у меня вечно со вторым законом ньютона проблемы, я никогда не вижу за счет какой силы появляется ускорение.
Аноним 23/10/16 Вск 16:12:06 #112 №394059 
>>394053
Я хуй знает, но по логике-то как раз получается, что сила трения никуда его не тянет, а сопротивляется успорению.
Аноним 23/10/16 Вск 17:20:21 #113 №394065 
>>394059
Я с тобой согласен, но что его тогда блять назад тянет? У меня были мысли, что мб сила трения второго тела тянет его назад, но это абсурд какой-то. Мб сила тяжести вниз, но тоже бред вроде как
Аноним 23/10/16 Вск 21:33:21 #114 №394084 
>>394065
Ни что, блять, назад не тянет. Он тупо хочет на месте остаться, потому и соскальзывает.
Аноним 24/10/16 Пнд 17:36:22 #115 №394143 
>>393781
Почему "масса ничтожно мала приравняли" к нулю?
Аноним 24/10/16 Пнд 18:19:47 #116 №394146 
Наукач, помню что читал где-то формулу какой-то постоянной, которая в моей памяти выглядит так:
Постоянная = Ошибка времени + ошибка пространства
Хуй знает что это и о чём, но в моей памяти это есть, нагуглить не могу.
Суть была в том, что ошибка времени постоянно растёт Со временем, лол, то есть ошибка пространства уменьшается на столько же, и в один момент ошибка пространства должна стать отрицательной и тогда что-то случится.
Помогите! Спать спокойно не могу.
Аноним 24/10/16 Пнд 19:21:37 #117 №394150 
>>390896 (OP)
https://www.youtube.com/watch?v=XqTLp7RxcbE

Аноним, заряд тут тратится, не?
Аноним 24/10/16 Пнд 22:09:38 #118 №394160 
14773361784080.jpg
Можно ли превратить энергию в материю?
Т.е. при взрыве атомной бомбы, например использовали 30 кг урана, тогда получается, что для получения 30 кг материи (в нашем случае урана) понадобится энергия атомной бомбы?
Аноним 24/10/16 Пнд 23:34:39 #119 №394176 
>>394160
>энергию в материю
Энергия и материя это разные вещи, но взаимосвязанные. Энергия - это мера движения и взаимодействия материи и мера для превращения одно формы материи в другую. Материя - это материи, её определения я не знаю. Голой энергии без материального носителя не бывает, как и материи без энергии, потому что всегда есть такая система отсчета в которой материи движется. Так что твой вопрос преозвучивается в "может ли одна форма материи превращаться в другую форму материи". Касаемо ядерного взрыва То там часть атомно-молекулярной формы материи превращается в материю в форме фотонов высокой энергии, высокой частоты, низкой длины волны, которых тут же поглощают атомы-молекулы-ионы-свободные электроны-альфа частицы-свободные нейтроны и разогреваются до ебический температур, а потом через время томы-молекулы-ионы-свободные электроны-альфа частицы-свободные нейтроны испустят часть поглощенных фотонов только другой длины волны/частоты/энергии а часть будет путешествовать в форме внутренней энергии из-за ударов атомы-молекулы-ионы-свободные электроны-альфа частицы-свободные нейтроны ядерного взрыва об другие атомы-молекулы-ионы обычного мира разогревая их. Что же касается формулы e=mc2, то масса не является хорошой и объективной мера количества материи. В современной физике масса это мера гравитационного взаимодействия и мера инертности. Т.е. есть гравитационная масса(пассивная и активная) и инерционная масса. Активная гравитационная масса - это мера того какую гравитацию создает тело(сколько испускает гравитонов или на сколько сильно искажает пространство-время в зависимости от какой теории смотреть ОТО/СТО или квантмех). Пассивная гравитационная масса - это мера того как тело поддается действию гравитации(как сильно поглощает гравитоны или подается искривлению пространства-времени). Инерционная масса - это мера инертности, т.е. имунитета/резиста к взаимодействию, например инертные газы, их так называют потому что они не особо-то химически взаимодействуют. А многие формы материи могут и не участвовать в гравитационном взаимодействии, например фотоны, у них нет активной гравитационной массы, но есть пассивная, поэтому они летают в искривленном пространстве-времени. Насчет инертной не помню вроде нету. По этому принято говорить что фотоны обладают эквивалентом массы из формулы e=mc2, но массы покоя нету.
Получается твой вопрос в следующем можно ли материю в форме фотонов превратить в материю в форме атомов/молекул. Ответ: Можно следующим образом, закачать дохрена энергии в один фотон и шандорахнуть его об что-нибудь, желательно в такой же фотон с такой же энергией, в резульате фотон(ы) распадуться на пары электрон-позитрон(относительно мало энергии. 1,022 МэВ если память не изменяет), протон-антиротон и нейтрон-антинейтрон(относительно много энергии) и через время протоны притянут электроны и врежутся в нейтроны(то же самое с позитронами, антипротонами и антинейтронами), а свободная часть нейтронов/антинейтронов через 15 минут распадется на протоны и электроны/антипротоны и позитроны и у тебя получиться газовая смесь из водорода(протия и дейтерия) и антиводорода(антипротия и антидейтерия). Дальше разделяй их и методом термоядрного синтеза как в звездах создавай из водорода нужные тебе элементы, а потом из них методами химии нужные тебе вещества, то же самое с антиматерией.
P.s. Насчет массы покоя и фотонов пусть меня поправят другие аноны, прост есть ощущение что написал не правильно или забыл написать.
Аноним 25/10/16 Втр 09:48:39 #120 №394211 
>>394160
Гипотетически - да.
Вблизи горизонта событий черной дыры могут появляться пары материальных частиц, одна из которых вылетает из черной дыры, а другая в нее падает.
но это не точно
Аноним 25/10/16 Втр 11:07:09 #121 №394214 
>>394176
>>сколько испускает гравитонов
Воу-воу, палехче! Квант гравитационного взаимодействия зафиксирован приборами?
Аноним 25/10/16 Втр 12:12:06 #122 №394216 
>>394160
При атомном боьулехе в энергию переходит всего нихуя ℅℅пара-тройка граммов℅℅ урана из всего заряда.
Аноним 25/10/16 Втр 18:20:15 #123 №394242 
>>394084
окей, тогда я не могу составить уравнение, не подскажешь как?
Аноним 25/10/16 Втр 18:22:44 #124 №394243 
>>394242
Да.
Аноним 27/10/16 Чтв 12:49:30 #125 №394457 
>>394214
Гравитационные волны зафиксированы, а гравитоны это очень короткие гравитационные волны.
Аноним 27/10/16 Чтв 13:42:31 #126 №394473 
Поясните на пальцах за скалярное поле.
Это типа эфира?
Аноним 27/10/16 Чтв 17:54:10 #127 №394516 
>>394473
Скалярное поле это математика. А эфир это физика. Ты понимай разницу.
Аноним 28/10/16 Птн 19:12:58 #128 №394628 
Как вообще определили существования горизонта событий черной дыры? Такой же вопрос насчет сингулярности.
Аноним 28/10/16 Птн 19:56:42 #129 №394630 
>>394628
математически. эти слова - части математической модели теории относительности. берешь, читаешь азы теории относительности и сразу все понимаешь.
Аноним 28/10/16 Птн 20:00:40 #130 №394631 
>>394473
это такое множество. как стадо баранов. только конкретно у этого стада есть свойства поля. а поле это понятие математики. на вике хорошо обьяснено.
Аноним 29/10/16 Суб 19:19:21 #131 №394706 
>>394053
физтеховская задачка первого курса. помню её. кубик что наверху толкает в бок сила трения, равная силе нормальной реакции опоры на мю. на кубик снизу действует сила что на веревке и противоположно направленная сила трения с бруском Ftr2. что непонятного?
Аноним 30/10/16 Вск 09:43:06 #132 №394754 
>>392523
Если рассматривать спин, как момент импульса, связанный с вращением, то 1/2. Если спин как спин, то максимальная сумма всех спинов
Аноним 31/10/16 Пнд 03:09:44 #133 №394833 
Наверное совсем глупый вопрос. Наша галактика называется Млечный путь, так? т.е. все звезды(или почти все?), которые я вижу на небе- часть этой галактики, так? Но название появилось еще с древних времен когда увидели на небе...... что? что конкретно видно на картинках где показывается "млечный путь"? по идее ведь абсолютно все, что я вижу на небе- это он и есть, а тут что-то типа особенное... не пойму нихера
Аноним 31/10/16 Пнд 07:00:35 #134 №394839 
>>394833
Направление в сторону центра выглядит как звездное скопление, проходящее примерно под углом 60 градусов в моих ебенях через небо. Вот это и есть Млечный Путь
Аноним 31/10/16 Пнд 08:41:47 #135 №394841 
>>394754
Че, блять? У меня на районе за такое убивают нахуй.
Аноним 31/10/16 Пнд 13:12:31 #136 №394860 
PBkF5eiH8NY.jpg
>>394833
>этот момент, когда начали задавать вопросы каторжане, из-за городской засветки не видевшие Млечный Путь.
Аноним 31/10/16 Пнд 18:51:22 #137 №394915 
>>394860
Как это я его не видел, когда я в нем живу? Его только слепой не видит.
Аноним 31/10/16 Пнд 21:15:33 #138 №394929 
>>394833
Сплошная полоса на небе белого цвета, отсюда Млечный Путь.
Аноним 01/11/16 Втр 00:41:57 #139 №394947 
>>394929
Странно. Никогда ее не видел. Ну, ладно в городе светло, но у бабушки в деревне(юг Воронежской области) тоже нифига вроде...
Аноним 02/11/16 Срд 00:06:24 #140 №395095 
Как так преподают физику, что сначала говорят, дескать, математика вам сугубо прикладная нужна для расчётов, а при изучении квантовой механики оказывается, что надо знать теорию, Гильбертовы пространства всякие. Получается, что математик при изучении заранее в более выигрышном положении оказывается за счёт более глубокого знания математики.
Аноним 02/11/16 Срд 12:46:34 #141 №395141 
>>395095
тем, кому преподают физику параллельно и математику преподают на уровне, достаточном чтобы гильбертовы пространства понимать.
Аноним 06/11/16 Вск 20:13:45 #142 №395535 
>>390896 (OP)
1. уже пояснили
2. галактика создает "поле", и это "поле" действует на Солнце, а не сама галактика.
Аноним 07/11/16 Пнд 16:37:31 #143 №395585 
>>394143
Почему это ничтожно мала? Как раз отсутствует.
Это, кстати, является одним из условий наличия возможности достижения телом или частицей скорости света.
Аноним 07/11/16 Пнд 16:38:46 #144 №395586 
>>394146
Неопределенность Гейзенберга.
Аноним 07/11/16 Пнд 17:16:29 #145 №395592 
14571681341842.jpg
Как работает ЭМИ? Не могу нигде найти адекватное разъяснение, разложенную по полочкам статью.
Везде все поверхностно, мол возникают токи в проводниках и т.д. Мне нужна именно физика процесса: вот э\м импульс, вот полупроводник, каким образом он на него воздействует, как образуются токи и т.д. Помогите найти хороший источник пожалуйста
Аноним 07/11/16 Пнд 18:33:57 #146 №395598 
>>395592
Так там наведённые токи всё нахуй сжигают, не?
Ну типа если проводник попадает в магнитное поле, то в нём ток появляется, вся хуйня.
Аноним 07/11/16 Пнд 19:46:50 #147 №395603 
>>393781
>>393781
>у света массы нет
дальше не читал
Аноним 07/11/16 Пнд 19:47:42 #148 №395604 
>>394146
>>394146
ты дебил. нахуй ты сюда пришел, если даже в преобразование фурье не можешь?
Аноним 07/11/16 Пнд 20:01:49 #149 №395606 
>>394176
>Голой энергии без материального носителя не бывает
любое излучение является чистой энергией
Аноним 07/11/16 Пнд 21:25:15 #150 №395610 
Как летает Martin Marietta X-24A?

Я правильно понимаю, что крылья нужны, чтобы давление воздуха снизу(сверху давление меньше из-за выгнутости верхней части) поднимало самолет вверх?
Аноним 07/11/16 Пнд 21:45:00 #151 №395612 
>>395606
>любое излучение
которое состоит из частиц(материальный носитель)
Аноним 07/11/16 Пнд 21:52:41 #152 №395615 
>>390896 (OP)
Как собираются писать программу и аллгоритмы для нанороботов? Там же виртуальная память будет маленькая(вангую максимум 10-100 байт), да и потом нужно будет устройство которое поймет информацию и начнет управлять нанороботом. По моему фигня эти нанороботы, я могу поверить в микророботов которые с размером с бактерий или клеток, но никак в роботов состоящих 100-1000 атомов.
Аноним 07/11/16 Пнд 22:48:43 #153 №395626 
>>395612
ты о чем? о том что фотоны материальны?
ну так и энергия материальна. у неё масса есть например. Еравноэмцэквадрат в обе стороны работает.
Аноним 07/11/16 Пнд 22:55:07 #154 №395627 
>>395626
>>395612
>>395606
Ебать филологи собрались.
Аноним 07/11/16 Пнд 23:01:45 #155 №395629 
>>395592
>Не могу нигде найти

>адекватное разъяснение,
>разложенную по полочкам статью.
Противоречие.
Адекватное разъяснение = законы электродинамики. Уравнения максвелла+материальные уравнения. Всего7 штук. Из которых можешь любую хуйню выводить, если достоин такими вопросами задаваться. Как это делает любой школьник-радиолюбитель.
Разложенная по полочкам статья= привлекающий дегенератов контент, создающий у читателя иллюзию ощущения понимания.

Даже сама постановка вопроса лульная
>>Как работает электро-магнитный И...(импульс, излучатель - похуй что ты там имел в виду)
Работает - это твой менеджер в макдональдсе.
А ЭМ-поле - распространяется.
Аноним 08/11/16 Втр 03:08:13 #156 №395642 
>>391465
Элементарно, военные давно уже батарейки делают на изотопах. У тебя бетта излучение - это поток электронов, альфа - поток ядер гелия. Ничто не мешает создать разницу потенциалов, что в конечном итоге и будет образовывать ЭДС, в космоаппаратах вроде плутоний 200 какой-то, или калифорний может. Наши запилили на никеле-63, он хуячит электронами, которые нужно только уловить, что не является проблемой, особенно магнитом.
Аноним 08/11/16 Втр 05:52:13 #157 №395669 
>>395629
Ну извини, ваннаби-жесткий-физик.
Мне нужно узнать о действии ЭМИ на полупроводники, как происходит процесс электрического пробоя. Думал, анон вспомнит парочку хороших авторов. Мне нужен принцип действия на словах, понимаешь?

Действительно, зачем ты высрал свое ненужное мнение?
Аноним 08/11/16 Втр 06:12:51 #158 №395675 
>>395669
Блять ты формулируй тогда задачу хотя бы. Научись хоть этому.
Пробой он и есть пробой. Пробивает. В норме электрон не может проскочить - на его пути полей, его отталкивающих. Не хватает ему скорости, не достаточно силы, протаскивающей его через диэлектрик.
Поддал напруги - выросло поле, его пихающее. Он и пробился. По пути расхуячил всё, столкнулся неоднократно, отдал часть энергии и сразу же из поля ее обратно хавает разгоняется и летит дальше устраивать пиздец. пробой он и на словах пробой.
Аноним 08/11/16 Втр 06:21:08 #159 №395676 
>>390896 (OP)
Вот говорят что звук медленнее света, но тогда почему люди не видят задержку когда разговаривают с друг другом, ведь судя по этой теории, слова должны чуть отставать от движении губ, я даже специально начал наблюдать за этим, когда разговариваю с продавцами в магазине, спрашиваю время у случайных людей, разных, чтобы исключить некоторые уникальные случаи когда у человек голос быстрый от природы, даже перед зеркалом издаю звуки, но никакой рассинхронности не наблюдается.
Аноним 08/11/16 Втр 06:50:25 #160 №395681 
>>395676
Да, да, а юный Ахилл не может догнать черепаху. А теперь подумай, почему ты сначала видишь вспышку молнии, а потом слышишь гром. Но можешь и ещё на рты продавщиц смотреть.
Аноним 08/11/16 Втр 07:00:21 #161 №395682 
>>395669
Действие ЭМИ на полупроводник такое же, как пробой конденсатора, полупроводник, диод, является в одном направлении проводником, а в обратном конденсатором. Правда с низкой ёмкостью, но всё же. Поскольку ЭМИ вызывает сильную разницу потенциала на контактах, то это вызывает пробой и окисление полупроводника, после чего он превращается в обычный диэлектрик, или испаряется. По сути у тебя вообще всё взаимодействует как полюса конденсатора, и при накоплении достаточного ЭДС, пробой будет в месте с наименьшим сопротивлением.
Аноним 08/11/16 Втр 07:18:10 #162 №395685 
>>395592
Тут без авторов всё понятно, у тебя когда ток идёт по проводу, возникает электромагнитная индукция вокруг него. Так же если вызвать изменение электромагнитного поля вокруг провода, то в нём начнёт течь ток. Смотри устройство генератора, трансформатора. ЭМИ распространяется со скоростью света как поле во все стороны, обычно как серия импульсов большой мощности. И проходя через метал ЭМИ заставляет в нём течь ток. В обычных проводах ток уходит по проводу с той же скоростью света, а вот конденсаторы, диоды, триоды, ток не проводят равномерно внутри себя, как обычный металл. И вот ЭМИ проходя как волна сквозь полупроводник, или конденсатор, вызывает резкое возникновение тока на одном из контактов. У конденсатора происходит переполнение ёмкости и пробой. У диода, если сначала волна эми достигла анода(с той стороны где диод ток проводит), то ток уходит к катоду спокойно. Но потом поле достигает катода, а на аноде уже тока нет, то возникает разница потенциалов на аноде и катоде диода, ЭДС проще говоря, и если ЭДС превышает допустимую нагрузку диода, то происходит пробой, с выжиганием или испарением диэлектрика.
Аноним 08/11/16 Втр 11:56:01 #163 №395733 
14571697290221.jpg
>>395682
>>395685
>>395675
Спасибо, анчоусы, теперь немного понятней ^^
Аноним 08/11/16 Втр 19:48:43 #164 №395852 
Можно ли создать краску, которая на солнце белая, а в белом светодиодном освещении черная?
Аноним 08/11/16 Втр 19:57:18 #165 №395857 
Как вычисляется индукционный ток в сверхпроводящем контуре?
Аноним 08/11/16 Втр 20:45:33 #166 №395862 
>>395852
прям совсем белая на солнце будет врядли, на смотреть спектральные характеристики, в принципе можно сделать что-нибудь с люминофором, который будет инициировать свечение краски от УФ, так же в краску намешать рассеивающих и отражающих частиц.
А для спектра от светодиода сделать полное поглощение, но я что-то сомневаюсь, что есть такие специфические материалы, чтобы задать чёткие условия отражения - поглощения. Можно посмотреть в сторону поляризации и дифракционных решёток, в общем сложные нано-полимеры, или типа того. То есть это будет не совсем поверхность, а некоторая объёмная структура, которая будет преломлять свет, поглощать свет, перераспределять внутри себя и испускать.
>>395857
наверное так же, как и в обычном, сила Лоренса от свойств токопроводящего материала не зависит, смотри закон Фарадея и уравнения Максвелла.
Аноним 09/11/16 Срд 11:56:49 #167 №395891 
>>395852
только если флоуресценция какая, лол. но нет.
максимум - серая, энергии не хватит
Аноним 09/11/16 Срд 23:44:10 #168 №395942 
123232.jpg
>>395852
например так:
Нужен не обычный белый диод
А добавить узкополосных фильтров, ужать его три полосы максимально. чтобы диод светил не тремя горбами а тремя палочками китайскими.
А краска должна на этих полосах поглощать, а остальное норм рассеивать.
и подогнать три компоненты эти так, чтоб был баланс, чтоб сохранялась пропорция излучения и поглощения для трех частот.

Идея пиздатая мне понравилась жги еще. я буду физику дописывать.
Аноним 10/11/16 Чтв 00:25:12 #169 №395944 
>>395942
с теорией то понятно, но боюсь интенсивность очень упадёт, на солнце белой не будет, будет серой, светило у тебя по всему спектру работает.
Аноним 10/11/16 Чтв 00:36:30 #170 №395947 
images[1]
Ну, у меня какбэ о физике, но и какбэ о математике.
В общем, читал я тут в толчке о топологии и непрерывных отображениях, и вот на моменте о неразличимости бублика и кружки у меня возникли вопросы.
1. Положим, вселенная имеет форму бублика, и некто, пусть будет боженька, деформирует ее в форму кружки таким образом, что бы это было непрерывное отображение. По идее, любая линия (как изгиб гитары ,так и всякие мировые линии) в изначальной вселенной сохранит непрерывность и в итоговой. Тогда получается, мы даже не сможем понять, что деформация вообще произошла, ведь с точки зрения непрерывности обе формы неразличимы? Или сможем? Как быть с изменившейся длиной линий? Время же тоже будет пропорционально деформироваться?
2. А если боженька вместо непрерывного отображения тупо пробьет в бублике еще одну дырку, тогда, по идее, мы сможем заметить это как странный пространственно-временной артефакт, т.к. произойдет разрыв и нарушение непрерывности множества линий? Допустим, дырень придется на корову, что получится, жопа с рогами или черная дыра (в ней ведь тоже происходит разрыв мировых линий, емнип)?
Аноним 10/11/16 Чтв 01:02:26 #171 №395955 
>>395944
почему сильно то, если ты узкие полоски вырезал из всего полотна? тем более белый-серый это всё относительно. в пасмурную погоду какой у тебя снег? сп пикрилейтед
да и вообще хули ты со своим "сильно" тут влез? бессмысленный кусок говна а не слово.
Аноним 10/11/16 Чтв 01:21:48 #172 №395956 
>>395947
Кружка и бублик неразличимы на уровне топологии, но на уровне, скажем, кривизны пространства вполне таки различимы. Если мы измеряем кривизну как гравитацию, то изменения заметим.
Аноним 10/11/16 Чтв 01:42:33 #173 №395959 
>>395956
Вот об этом я не подумал. С другой стороны, это же астрофизический масштаб, мы еще миллиарды лет не будем подозревать об изменении кривизны пространства...
Аноним 10/11/16 Чтв 15:08:16 #174 №395984 
светодиоды.png
Spectra.Classical&Function.jpg
>>395955
это ты влез, я до тебя ответил значительно точнее и информативнее.
"сильно" где ты это слово у меня прочитал, ты жопой читаешь? Назови мне химические соединения с узким отражающим спектром, назови мне светодиоды с узким изучающим спектром, ты ж ёбаный диванный кукаретик.
>снег
Снег вообще цвета не имеет, кристаллы воды абсолютно прозрачны, снег испускает преломлённый свет, а не отращённый... хули ты вообще пасть свою разинул, неуч?
Аноним 10/11/16 Чтв 15:11:37 #175 №395985 
>>395947
Ты диф.математику знаешь? Почитай что Перельман доказал. Вселенная никакой не бублик, так что и говорить не о чем.
Аноним 10/11/16 Чтв 15:24:19 #176 №395987 
>>390896 (OP)
Ну гравитация и магнитные поля распространяются со скоростью света.
Аноним 10/11/16 Чтв 17:27:20 #177 №395995 
Всем известно, что вселенная содержит определенный набор фундаментальных частиц, имеющих различные характеристики и свойства, порождающих атомы и молекулы, или существующих сами по себе. И ученые в адронном коллайдере разгоняют более крупные частицы, сталкивают их друг с другом, получая более мелкие, но всё еще входящие в тот уже известный всем нам сет. Эти частицы - сгустки энергии, да что говорить, всё во вселенной - сгустки энергии определенной плотности и энтропии. И теперь собственно вопрос. Есть ли теоретическая возможность, имея полные знания о сущности энергии, создать новую элементарную частицу, никогда до этого не существовавшую ирл?
Аноним 10/11/16 Чтв 19:37:00 #178 №396001 
>>395985
Ну я имел ввиду как если бы вселенная была бубликом, я же не утверждаю, что она на самом деле бублик. И причем тут диф.математика? Какой Перельман, Яков или Григорий?
Аноним 10/11/16 Чтв 19:56:10 #179 №396005 
img-19c46.jpg
>>396001
Гришка конечно, он топологическую задачу дифматом решил, кек, которою топологи обычно не знают/не понимают. Собственно из-за чего Гриша и взбугуртнул.

Раз ты допустил тороидальную форму, то допускай и всё остальное уж, будь плохим парнем.
Аноним 11/11/16 Птн 16:52:57 #180 №396043 
Анон, в статье вики про Большой взрыв сказано: "Ранняя Вселенная представляла собой высокооднородную и изотропную среду с необычайно высокой плотностью энергии". Но занимает ли энергия какое-либо пространство? Если нет, то откуда тогда понятие "плотность энергии"? Ведь плотность определятся относительно какого-либо пространства?
Аноним 11/11/16 Птн 17:02:31 #181 №396044 
>>396043
"Ранняя вселенная" не значит "до возникновения вселенной"
Аноним 11/11/16 Птн 17:06:42 #182 №396046 
>>396044
А кто сказал, что значит? Я понял это так, что после взрыва образовалось некоторое пространство, которое представляло из себя ту самую изотропную среду, которая была заполнена высокоплотной энергией. Но ведь энергия - это мера. Типа как длина. Как тогда она может занимать пространство? И откуда у неё плотность?
Аноним 11/11/16 Птн 17:17:04 #183 №396050 
>>396046
Ты бы еще спросил про плотность тока.
Аноним 11/11/16 Птн 18:52:56 #184 №396066 
>>396050
не вижу никаких проблем с плотностью тока. постоянный ток течет по всему обьему проводника, а вот высокочастотный - чем выше частота, тем ток ближе к поверхности проводника, а в центре его уже нет. + течь может 3 электрона паралельно, а может 350.
>>396046
>Как тогда она может занимать пространство? И откуда у неё плотность?
сравни два кубических сантиметра пространста рядом с собой, где нибудь на пол пути к солнцу от земли и в центре солнца.
наверное поймешь что да как :3
Аноним 11/11/16 Птн 19:20:16 #185 №396069 
14771719079900.jpg
Измеряемая в кельвинах температура не более чем показатель колебаний атомов.
А что будет, когда случается абсолютный ноль с физической точки зрения с атомом?
А что будет, если мы возьмём температуру, например, -50 кельвин? Ясное дело, атомарный вопрос отпадает ещё на нуле кельвин, тут вопрос в другом - можно ли получить температуру кристаллизации или хотя бы сжижения пространства (времени там, гравитации, электро-магнитных полей)
Аноним 11/11/16 Птн 20:20:00 #186 №396081 
Screenshot20.png
>>395984
еблан-теоретик мамкин
>назови мне назови плиз кококо я не знаю как перейти от умных слов к эксперименту но я умненький читал много слов знаю наноматериалы кококо
На лабы надо было ходить.
Берёшь светодиод с горбом обычный
Берешь УЗ-КО-ПО-ЛОС-НЫЙ фильтр, лепишь на светодиод. ВУАЛЯ блять - "светодиод с узким излучающим спектром". Магия уровня сцы.


>Назови мне химические соединения с узким отражающим спектром
Сам догадаешься как такое осуществить.

Перечитал внимательней твой лел "точный и информативный" пост.
Проиграл с вонаби-физика-эксперта.
>это ты влез, я до тебя ответил значительно точнее и информативнее.
Прямо вижу слёзы омегана-википидора, который плачет из-за того, что кто-то на анонимной борде отнимает у его жалкого эго статус копчёного.
>но я что-то сомневаюсь, что есть такие специфические материалы, чтобы задать чёткие условия отражения - поглощения.
>Можно посмотреть в сторону поляризации дифракционных решёток
Лол. сцы-петух лепит "умные слова" не в тему, думая кому-то запудрить мозг.
>в общем сложные нано-полимеры, или типа того
>поляризации, дифракционных решёток
>в общем сложные нано-полимеры, или типа того

Пшел нахуй уебище чубайсовское.
Аноним 11/11/16 Птн 20:26:09 #187 №396082 
Screenshot21.png
>>395984
>Снег вообще цвета не имеет
С этого тоже проиграл. Лос-стрик прямо с тебя поймал.
Твоя мимикрия под иксперта в оптике похожа на видрелейтед брюса ли - https://www.youtube.com/watch?v=WrcmxgKcuEw
Снег имеет цвет. Цвет это именно цвет, еблан. Субъективная характеристика зрительного восприятия.
На солнце снег -белый. Если нассышь на него - он желтый. Посветишь в темноте зеленым фонариком - будет зелёный. Способен понять?
Субъективное от объективного отличаешь - или ты шизоид?
Аноним 11/11/16 Птн 20:29:33 #188 №396083 
>>396069
нет смысла говорить о температуре отдельного атома.
но если ты берешь тело из множества атомов и допустим охлаждаешь до нуля - атомы всё равно колеблятся. есть некий минимальный уровень энергии колебаний атомов в твердом теле. "остановить" колебания полностью нельзя типа.
>хотя бы сжижения пространства
Спроси у нанополимерного клоуна. Он в таких делах наверняка эксперт.
Аноним 11/11/16 Птн 20:31:15 #189 №396084 
>>396069
При нуле Кельвин атом/молекула не будет колебаться, а при -50 Кельвин то же самое, но для того чтобы заставить его колебаться при нуле кельвин будет достаточно просто пнуть, а при -50(или -икс) придется потратить тепла больше(теплоемкость вещества умножить на массу вещества умножить на 50(икс)). Но это невозможно так как принцип неопределенности Гейзенберга и атом всегда будет колебаться и всегда будут системы отсчета в/относительно которых атом/молекула будет двигаться, поэтому абсолютный нуль не достижим.
Аноним 11/11/16 Птн 20:37:39 #190 №396086 
>>395944
>светило у тебя по всему спектру работает.

Если б глядел пикрелейтед поста на который отвечаешь и знал хоть что-нибудь то распознал бы на пике под номером (1) спектр солнечный. Учит он меня блять как солнце светит лол.
Аноним 11/11/16 Птн 20:54:17 #191 №396087 
Окей, как будут слышны электромагнитные волны по струне (любой), принятые в приёмник.
Аноним 11/11/16 Птн 21:17:15 #192 №396090 
Аноны, есть здесь работающие репетиторами по физике? Как устроиться, ну или хотя бы найти пиздюков даунов, чтобы их обучать?
Аноним 11/11/16 Птн 22:24:50 #193 №396097 
>>396081
>а в белом светодиодном освещении черная?
читай задачу
>УЗ-КО-ПО-ЛОС-НЫЙ фильтр
примеры, спектральные характеристики? И ты собираешься учить физика - фотографа с 10+ стажем? Пиздец конечно, перейди от теории к практике, школяр.
>Сам догадаешься как такое осуществить.
Нет, не догадаюсь, научи?
>лел Лол.
Смешно дураку, что уши на боку.
Аноним 11/11/16 Птн 22:31:51 #194 №396099 
hli10.png
>>396082
>Снег имеет цвет. Цвет это именно цвет
>Субъективная характеристика зрительного восприятия
>Субъективная
а ты тралл конечно, "ты что дальтоник, скрипач, зелёный от оранжевого отличить не можешь"
Вообще-то цвет имеет физическое измерение, точное, блядь, абсолютное, а не субъективное.
Аноним 11/11/16 Птн 22:36:12 #195 №396100 
>>396086
>пикрелейтед
>детские каляки малаяки
https://2ch.hk/sci/src/390896/14787242500500.jpg
или
https://2ch.hk/sci/src/390896/14787796964801.jpg
что ты диванный, я давно уже понял
Аноним 11/11/16 Птн 23:01:25 #196 №396103 
>>396099
>Вообще-то цвет имеет физическое измерение, точное, блядь, абсолютное
школьник, плиз.
Аноним 11/11/16 Птн 23:26:03 #197 №396107 
>>396103
λ
Аноним 11/11/16 Птн 23:42:00 #198 №396108 
>>396069
> А что будет, если мы возьмём температуру, например, -50 кельвин?
В некотором смысле можно охладить ниже абсолютного нулю. Тут понимается инверсия распределения частиц по энергиям. Успешные эксперименты уже были, гугли.
https://ru.wikipedia.org/wiki/Отрицательная_абсолютная_температура
https://lenta.ru/news/2013/01/04/subzero/
Аноним 12/11/16 Суб 00:15:06 #199 №396110 
>>396107
Длина волны это длина волны, а цвет - это разговорное её название. И если оранжевый при ярком освещении назовут жёлтым, появляется Физическая Полиция со спектроскопом.
Аноним 12/11/16 Суб 00:36:51 #200 №396112 
>>396110
1. Назови-ка мне длину волны коричневого цвета.
2. Поцчему при смешении синего и желтого получается зеленый, если в смеси нет "зеленых" длин волн?
Аноним 12/11/16 Суб 01:26:35 #201 №396117 
>>396112
Не лезь в залупу, няша.
Аноним 12/11/16 Суб 01:45:32 #202 №396118 
>>396117
Нет ты.
Аноним 12/11/16 Суб 02:29:34 #203 №396121 
Я вот тут про тяжелую воду читал, и подумалось. Если легкая вода захватывает нейтроны, то должна превращаться в смесь разных изотопных вод, полутяжелой, тяжелой, etc. Причем, большинство изотопов водорода и кислорода быстро распадаются, и итоговая смесь лишь немного будет загрязнена тритием. Так зачем такие сложности с обогащением тяжелой воды, если ее можно получать для тяжеловодных реакторов с помощью обычных легководных, лишь потом дообогатив?
Аноним 12/11/16 Суб 02:38:51 #204 №396122 
>>396097
Регулярно отвечаю тут на вопросы про оптику и похожее, думал и тебе ответить, но ты настолько тупое говно, что решил просто на голову тебе насрать.
мимо-настоящий-физик
Аноним 12/11/16 Суб 02:39:51 #205 №396123 
filter6.jpg
>>396110
>Длина волны это длина волны, а цвет - это разговорное её название.
Не можешь разглядеть разницу между цветом и длиной волны.
>Вообще-то цвет имеет физическое измерение, точное, блядь, абсолютное, а не субъективное.
Точное "физическое измерение" имеет масса твоей жирной мамаши: 1488.228Е1337 кг.
>примеры, спектральные характеристики? И ты собираешься учить физика - фотографа с 10+ стажем?

Ты мерзкий даун. Умри.
Остальным анонам, пострадавшим от шизофазии копрофотографа:
http://golddragonoptics.ru/uzkopolosnye-filtry.html
Аноним 12/11/16 Суб 12:29:35 #206 №396138 
>>391226
> Например, от нас до галактики Андромеды она долетает за секунды.
Что за бред. Интересно, как ты посчитал эти секунды?
Аноним 12/11/16 Суб 19:56:35 #207 №396167 
1. Принцип неопределенности Гейзенберга. Поясните, этот принцип влияет только на наблюдателя (т.е. не даёт ему точно измерить обе величины) или же он постулирует, что никакие частицы/поля не имеют обеих точных величин?
В первом случае детерминизм еще не накрывается медным тазом, а просто становится неподконтрольным для наблюдателя. Во втором же всё реально по пиздец идет и вселенная рулетки крутит.
Аноним 12/11/16 Суб 20:28:27 #208 №396179 
>>396167
Ну не то, что бы не имеют обеих точных величин, а значения величин находятся в суперпозиции. А что это значит - есть разные интерпретации.
Аноним 12/11/16 Суб 20:36:20 #209 №396180 
>>396167
Есть 2 аспекта этой "проблемы" насколько я понял после 3х курсов, паря себе мозг этим вопросом с самого начала. Хотя я хуево учился так что можешь скипнуть.
1й- как по-факту обстоит дело для нас.
2й- как на самом деле?

Для нас точные измерения невозможны по принципиальным соображениям. Если опуститься на масштабы атомов - там нету никакого света. Измерения происходят путем тыканья в объект другим объектом. Тыканье передается посредством ЭМ/сильного поля. Ну ты понял - твердых тел то реально не существует, нету в реальности "касания" одного атома другим. Есть их взаимное отталкивание/притяжение за счет полей.
И вот ты взялся измерять скорость/координату частицы. Для этого надо в нее ткнуть условным прибором. Измерить отклик и сделать вывод. Залупа в том что в момент когда ты тыкаешь в частицу - ты сам на нее влияешь, изменяя ее импульс и соответственно координату.
Пример уебищный но позволяет прочувствовать ситуацию вроде. представь что ты с закрытыми глазами в темноте пытаешься точным динамометром "померять" падающие снежинки. Ты тычешь динамометром в воздух, попадаешь в снежинку и получаешь в ответ воздействие - де-факто это не одно число, а функция силы от времени.
Видишь на дисплее динамометра этот "импульс силы". От чего он будет зависеть? Во-первых от массы снежинки, во-вторых от ее скорости в момент столкновения. Тяжелая но медленная снежинка даст грубо говоря такую же кривую как легкая но быстро движущаяся снежинка. Ты в итоге точно не знаешь ни ее скорость ни массу - ни до "измерения" ни после.
Теперь представь что твой динамометр сам из ебаных снежинок состоит, эдакая хлипкая колеблющаяся конструкция из снежинок блять. Ведь дело происходит не при абсолютном нуле - всё дергается туда сюда.

Или хуй с ними со снежинками. представь что ты фотограф и фоткаешь футбольшый матч - следишь за мячом. УДАР - ты делаешь снимок. Показываешь редактору. Ты - измерительный прибор, мяч - частица а редактор - это физик.
Если выдержка маленькая - на фотографии четко виден мяч, его координата. Но по фотографии ты не можешь никак судить о скорости, если мяч на фото НЕ РАЗМЫТ. Ни величину ни направление скорости ты на фото не видишь.

Если же он размыт - ты, зная выдержку,теперь можешь судить о скорости. Но на фото уже будет не объект "мяч" а некое размытое длинное пятно. Причем ты не знаешь с какой стороны он летел - слева направо или справа налево - поэтому по этой размытости не можешь судить о том, в левой части "пятна" находится мяч на момент закрытия диафрагмы или в правой.

То есть эта неопределенность - не какая-то хрень из микромира, это свойство нашего мира, в котором всё происходит за счёт полей. Типа при большой выдержке волна светового поля от мяча размазалась по матрице фотоаппарата.


Второй аспект - это то о чем срались батьки физики когда ковался квантмех в 30-40 годах. Есть ли у частиц все характеристики ДО измерения - или они возникают в момент измерения? Есть ли "скрытые параметры"?
Срались они срались и ни к чему не пришли.
Потом в 50-60х какой-то перец по фамилии Белл придумал и строго математически обосновал эксперимент, проведя который, можно было бы сделать однозначный вывод по проблеме. Проверить т.н. неравенства Белла - выполняются они или нет. Если выполняются то частицы - чисто случайные звери. Детерминизм однозначно НА САМОМ ДЕЛЕ сосёт хуяку. Суть "неравенств" - подчиняется ли определенное поведение частиц статистике, в которой фигурирует 2 числа. 2/3 и 3/4. Если реальное число меньше 2/3 - то детерминизм сосет. Если больше - не сосет. Или наоборот - я забыл уже нахуй, думать лень, но суть не в этом. Суть в точности и простоте эксперимента.
Но эксперимент провести тогда было сложновато с точки зрения техники.
А потом толи в 80 толи в 90х наконец начали эксперименты такие проводить и получать выводы. Они продолжаются до сих пор - чтобы уменьшить сомнения, повысить точность вывода. Что "число" лежит по одну сторону от этого интервала. Но уже давно всем похуй потому что с достоверностью овер 99.999999% неравенства выполняются. Детерминизм давно под шхонкой петушится. Такие дела.
гугли неравенства белла.
Аноним 12/11/16 Суб 20:40:51 #210 №396182 
>>396167
> Во втором же всё реально по пиздец идет и вселенная рулетки крутит.
Вообще "пиздецом" ты это называешь чисто из традиционных соображений. На самом деле по прошествии времени мне комфортнее стала картина с рулеточкой. Швабодка и всё такое.
А без рандома - у тебя есть сраная судьба со сраной однозначной траекторией и какие кренделя ни крути - один хуй с рельс не сойдешь.
Аноним 12/11/16 Суб 21:20:49 #211 №396188 
>>396167
>он постулирует, что никакие частицы/поля не имеют обеих точных величин
он постулирует что они не могут быть измерены одновременно.
Аноним 12/11/16 Суб 22:26:48 #212 №396196 
>>396182
Но ведь тогда получается, что и боженька у нас случайный!
Аноним 12/11/16 Суб 23:15:38 #213 №396199 
Почему солнце 6к градусов бело-желтое, тогда как железо в 1500градусов уже тоже почти белое?
Аноним 12/11/16 Суб 23:23:10 #214 №396200 
>>396199
Солнце херачит через атмосферу, а любая мелкая примесь (и просто флуктуации давления) рассеивает синие волне сильнее, чем красные. Поэтому в тумане или на закате солнце аж красное а не просто желтое. По идее, чем выше залезаешь, тем белее должно казаться.
Аноним 12/11/16 Суб 23:41:05 #215 №396205 
>>396196
естественно - триединство, слышал?
Аноним 12/11/16 Суб 23:48:58 #216 №396206 
>>396182
Это вопрос не имеет прямого отношения к физике, он философский
Аноним 13/11/16 Вск 08:20:28 #217 №396249 
1. Правда, что в центре нюки температура 10лярдов градусов? Сколько по времени это длится?
2. Скорость разлёта продуктов взрывчатки километры в секунду, значит, если б вместо кинетической энергии была одна тепловая (например, взрывчатку заперли в неразрушимом прозрачном ящике), температура будет дохуя сотни тысяч?
3. Говорят, типа самая высокая температура в коллайдере 10^12К и многие журнашлюхи считают её самой высокой, но из космоса прилетают звери куда большей энергии, чем протоны в кольце; даже с учётом большей протяжённости ускоряющего поля плотность энергии должна быть куда выше, указывает ли это на более высокотемпературные процессы?
Аноним 13/11/16 Вск 08:26:13 #218 №396250 
И да, сколько оборотов делают протоны в Баке, прежде чем столкнуться? Один, несколько или очень много?
>>396249
Аноним 13/11/16 Вск 09:16:50 #219 №396251 
111.jpeg
Аноним 13/11/16 Вск 12:57:29 #220 №396257 
>>396043
почему расплавленный полиэтилен прозрачный как стекло, при остывании тускнеет?
Аноним 13/11/16 Вск 16:20:27 #221 №396274 
>>396179
>>396180
>>396182
>>396188
Так и не понял, величины реально не имеют одновременно двух точных значений или только наблюдатель никогда не сможет их измерить, а на самом деле они есть? Или у физики нет точного ответа на этот вопрос.
Аноним 13/11/16 Вск 16:27:01 #222 №396275 
>>396249
Для неравновесных процессов к температуре надо относится осторожно. Если вычислять по формуле Планка, для гелий-неонового лазера, выйдут совершенно бешеные температуры. И к коллайдеру понятие температуры тоже не применимо - там равновесием и не пахнет. Или если при распаде ядра получается дохера энергичная частица, интерпретировать энертгию как кТ не всегда имеет смысл.
Аноним 13/11/16 Вск 23:39:01 #223 №396304 
>>396110
Даже с условностью цветовосприятия передача одного оттенка ограничена некоторым конечным множеством интенсивности волн. Ты не сможешь получить красный канал из зелёного и синего - никогда.
>>396112
>Назови-ка мне длину волны коричневого цвета.
там есть чёткая ачх и спектральное разложение, даже условные речевые обозначения упираются на чёткие физические значения. Да ты можешь выразить цвет несколькими спектрограммами, но их набор конечен. Так же "коричневый", как и любой другое слово, может быть обозначением некоторого множества цветов. В отличие от белого или тем более чёрного.
>Поцчему при смешении синего и желтого получается зеленый, если в смеси нет "зеленых" длин волн?
потому что колбочки в глазу человека восприимчивы к спектральному диапазону, и мозг выполняет аппроксимацию между каналами. Зелёный свет стимулирует рецепторы так же, как жёлтый и синий.
>>396123
теперь для остоёбов и ебанько, специально повторяю как звучал вопрос:
>>395852
>Можно ли создать краску, которая на солнце белая, а в белом светодиодном освещении черная?
Звучал он так, и никак иначе, вопрос был прокраску, а не про светодиод.
Под своим ответом >>395862 я предполагал, что сравнение будет проводится с обычными белой и чёрной краской. Если ты сделаешь краску, которая, например, поглощает: красный, зелёный, синий - но отражает: фиолетовый, жёлтый и голубой, то рядом с обычной белой краской она будет серой, так как обычная белая будет отражать: красный, зелёный, синий, фиолетовый, жёлтый и голубой, а твоя только половину.
Твои ёба http://golddragonoptics.ru/uzkopolosnye-filtry.html узкополосные фильтры добавленные в краску или к источнику света дадут тебе чёрный цвет, блядь, потому, что ты долбаёб - теоретик, и физики процесса не понимаешь. Именно поэтому и существует байеровская матрица в фотоаппаратах. Я молчу о светопотерях на стыках сред.
>Можно посмотреть в сторону поляризации и дифракционных решёток
>сцы-петух лепит "умные слова" не в тему, думая кому-то запудрить мозг.
Ты даже не представляешь как работает мультипросветление и зачем просветление оптики нужно.
А теперь покажи мне фильтр для 3-х узких каналов, или завали ебало. Тебе придётся или делать сборку из монохромных светодиодов, а такого вопроса не стояло, или сплавить все фильтры в один, и у тебя получиться по 33% для каждого канала, по интенсивности, если это вообще возможно, и у тебя получится не белый, а очень и очень серый.
А вообще вкус устриц лучше обсуждать с теми, кто их ел, а не с долбаёбами-кукаретиками.

Аноним 14/11/16 Пнд 00:58:00 #224 №396306 
>>396304
Конченный.
>а в белом светодиодном освещении черная
>придётся или делать сборку из монохромных светодиодов, а такого вопроса не стояло
В твоем вопросе нет уточнения про то, какой конструкции белый светодиод - люминофорный или трехканальный. Трехканальный из коробки это есть сборка, питушиное ты рыло.
>узкополосные фильтры добавленные в краску
Еблан сука жертва аборта, какие фильтры в краску, хватит фантазировать.
>к источнику света дадут тебе чёрный цвет,
Шта ты несёшь глупец
>Если ты сделаешь краску, которая, например, поглощает: красный, зелёный, синий - но отражает: фиолетовый, жёлтый и голубой, то рядом с обычной белой краской она будет серой, так как обычная белая будет отражать: красный, зелёный, синий, фиолетовый, жёлтый и голубой, а твоя только половину.
А ну понятно.
ПТУшник не может в спектры.
поглощается у него "красный".
То есть вони столько тут нагенерил, а оказалось, что ты просто не понимаешь нихуя.
Не понимаешь, что в красной области один материал может поглощать горб полуширины 100нм, а другой - 10нм. В твоих ПТУшных терминах - это одно и то же.
не понимаешь, что "обычная белая краска" - это материал, спектр которого ОЧЕНЬ примерно ровный, и допускает либо неглубокие провалы либо узкие вырезы, которые ты своим интегральным оптическим прибором не заметишь.
Не понимаешь, что если из спектра отражения белой краски вырезать три узеньких полосы, то краска так и останется белой, если размер и положение вырезанных полосок правильно подобрать с учетом физиологии зрения. Ну хули, это всё СЛОЖНА для ПТУшника. СЛОЖНА БИЛЯ НЕ ПОНИМАЮ ДАЙТЕ КРАСКУ НЕ ХОЧУ ДУМАТЬ Я ФОТОГРАФ СВАДЬБЫ ДНЮХИ СЕЛФИ 83р за час работы макакой с зеркалкой!
я то привык что такие концепты даже дауны на потоке проглатывали..
>Именно поэтому и существует байеровская матрица в фотоаппаратах.
проиграл с твоего "поэтому". впрочем над даунами без умения строить выводы смеяться грешно лол.
терминами своими говнофотографов тут козырит.
я честно говоря термина байероская матрица не знал, потому что это очевидный способ записи цвета на матрицу. прикинь - если ты немного шаришь оптику и физику в целом, то тебе нахуй не нужно спец.терминов для понимания структуры и принципов работы любых приборов. достаточно знать что на входе и что на выходе. это и есть физическое прикладное образование, которое ты проебал.
>А теперь покажи мне фильтр для 3-х узких каналов, или завали ебало.
таких не видел. такой и не нужен. нужен трехкомпонентный светодиод, на каждый элемент которого по отдельности ставится по фильтру. хотя дауну хоть ссы в глаза всё божья роса. раз нету понимания спектров позвоночником за 10 лет опыта - ничего уже не поможет.

Аноним 14/11/16 Пнд 01:03:00 #225 №396307 
>и у тебя получиться по 33% для каждого канала, по интенсивности, если это вообще возможно, и у тебя получится не белый, а очень и очень серый.
Признайся что ты просто тупой чмошник без воображения и закончим на этом.
Что светодиод может давать больше потока чем солнце - и даже при вырезании 90% краска в свете светодиода будет казаться белой, а на солнце - серой. Что эта разница будет зависеть не только от того, сколько схавали фильтры, но от соотношения потока от солнца и светодиода в конкретных ситуациях и от окружающих предметов. Что "белый" и "серый" - это один и тот же цвет по дефолту, и суть не в цвете а в количестве фотонов и не только.
Но ты не осилишь , 100%, щас начнешь опять плакать. гурман устричный лол.
Аноним 14/11/16 Пнд 01:34:40 #226 №396308 
>>396274
они имеют значения, но нельзя одновременно померять потому как процесс измерения вносит ошибки.
Аноним 14/11/16 Пнд 01:47:12 #227 №396309 
>>396306
>В твоем вопросе нет уточнения про то, какой конструкции белый светодиод - люминофорный или трехканальный.
Раз уточнения нет, значит решение должно быть универсальным или два решения. Но вопрос не мой.
>какие фильтры в краску, хватит фантазировать.
Эмульсия или лаки, ты вообще что-нибудь о красках знаешь?
>к источнику света дадут тебе чёрный цвет,
>Шта ты несёшь глупец
Ты уточни уже наконец, мне с тобой физическими терминами говорить (отсутствие фотонов с частотой эм колебаний в видимом спектре) или условное человеческое цветовосприятие, хули ты шалава жопой виляешь и лезешь в семантику?
>вырезать три узеньких полосы
откуда ты узенькие полоски взял? откуда твою мать ты их высрал? вопрос был про краску, оставь светодиоды в покое. Возьми тогда не горящие светодиоды, тогда любая краска чёрной будет, тупой уебан.
>концепты
Кукаретик как он есть. Не надо мне рассказывать теорию, ты её нихуя не знаешь.
Ты абсолютно не понимаешь, что человек так же воспринимает и чб цвет, отдельными рецепторами, и что у человека есть аккомодация, и что любое аналоговое восприятие строится на отношении сигнала к шуму, поэтому ты и отличаешь тона по яркости.
>Признайся что ты просто тупой чмошник без воображения и закончим на этом.
я признаю, что я говорю по делу, а ты живёшь в своём манямирке.
А теперь говноед, пофантазируй, с узкополосными фильтрами, если у тебя один единственный светодиод.
Аноним 14/11/16 Пнд 02:02:57 #228 №396310 
>>396307
ты не понял, тупой еблан, обычная белая краска под светодиодом, свет от которого пропустили через комбинированный фильтр из трёх узкоканальных, вообще ничего не отразит, так как канал пропущенный первым фильтром поглотят два других (уверен, что ты ни фильтры не использовал ни разу в жизни, ни диффузные гели), но если фильтры сделать тоньше, чтобы они пропускали часть света за приделами полосы пропускания, после 3 фильтров, это 6 переходов из среды в среду, у тебя в лучшем случае 25 процентов от общей интенсивности только в каждом канале останется, а поскольку интенсивность обратнопропорциональна площади, то падение яркости будет по параболе. Те есть чтобы увеличить яркость вдвое, нужно количество света возвести в квадрат.
Короче, продолжай дальше пребывать в своём манямирке.
Аноним 14/11/16 Пнд 02:49:45 #229 №396314 
>>396274
И то, и то. При чем Гойзенберг изначально придумал второй случай, немного думая о первом. При чем сделал это с ошибками. Но с тех пор много времени прошло и теперь, грубо говоря, есть два принципа неопределенности. Только в школе этому не учат, а как ни странно, даже в вузиках. Даже в википедии про это ни слова, лол.

>>396310
>Q: Если ты сделаешь краску, которая, например, поглощает: красный, зелёный, синий - но отражает: фиолетовый, жёлтый и голубой, то рядом с обычной белой краской она будет серой, так как обычная белая будет отражать: красный, зелёный, синий, фиолетовый, жёлтый и голубой, а твоя только половину.
>A: ПТУшник не может в спектры.
>Q: тынипонял, свет от которого пропустили через комбинированный фильтр из трёх узкоканальных, вообще ничего не отразит, так как канал пропущенный первым фильтром поглотят два других
Какой-же ты таки тупой и необучаемый дегенерат. Пошел фторой круг "я нимагу в спектр"

>Звучал он так, и никак иначе, вопрос был прокраску, а не про светодиод.
ПТУшник не может и в нотч-фильтры. Слова такого наверное и не слышал. Дальше подсказывать не буду.

%мимо-не-сдержался-и-ответил-дегенерату%%
Аноним 14/11/16 Пнд 03:13:21 #230 №396315 
>>396314
Хотя не, не нужны нотч-фильтры. Все проще.

От пригорелости с тупости птушника аж разметку проебал
Аноним 14/11/16 Пнд 03:32:17 #231 №396317 
>>396314
>нотч-фильтры
тупой еблан думает, что частота испускаемого электронами фотонов зависит от частоты тока в проводах. Пиздец, хули я на тебя время трачу, иди каляки маляки рисуй дальше.
Аноним 14/11/16 Пнд 05:38:31 #232 №396320 
>>396317
Боже какой даун, я чуть не обосрался от смеха. При чем здесь ток, дебилика? Мы про оптические фильтры тут вроде бы говорим. Вбил в гугл нотч и оно тебе электросхему показало? Или у фотографа в голове не укладывается, что можно оптический заграждающий фильтр/напыление сделать?
https://www.thorlabs.com/newgrouppage9.cfm?objectgroup_id=3880

Кстати, я не калякин, я этот >>396122
Калякину памятник надо ставить за терпимость в дискуссии с дегенератами.

Ответ на твой изначальный вопрос: да, можно, но ты этого никогда не сделаешь из-за отсутствия ресурсов, в первую очередь - мозгов.
Аноним 14/11/16 Пнд 06:03:27 #233 №396322 
>>396310
Хули ты вообще влез кадавр слепой блять из подзалупного творога в спор господ о проблемах прикладной науки?
встань со стула, сделай шаг назад, наклонись и начни сосать сам у себя

речь шла о трехкомпонентном светодиоде, никаких комбинированных фильтров
в то время как фотограф-свадебщик просто съехал с темы на ходу изменив условия задачи, отбросив одни и добавив другие, у него теперь блять светодиод обязательно люминофорныйй (хуй пойми чего он до него доебался если задача решаетя тройным белым диодом) делая вид что так и положено поступать человеку а не геммороидальной выдрочке.
Аноним 14/11/16 Пнд 06:04:05 #234 №396323 
>>396310
Блять даун ебаный какие то переотражения 6 переходов ПОЛЗИ НАХУЙ ОТСЮДА СЛЕПОЙ ОПАРЫШ СУКА
Аноним 14/11/16 Пнд 06:08:59 #235 №396324 
>>396309
Блять ну сколько раз тебе говорить уёбище ты тупое
>Эмульсия или лаки, ты вообще что-нибудь о красках знаешь?
Говно с мочёй с соплями как основной наполнитель твоего черепа - ты что-нибудь вообще о них знаешь?
Речь не о том что такой краски нет а о том что КРАСКА С "фильтрами" ни причем, она не фигурирует нигде и фильтры в краске не нужны. ПОНИМАЕШЬ
фильтры на светодиоде
перечитай
фильтры НА све-то-ди-оде
уебан.
а краска не с фильтрами а со спектром имеющим три выреза.
эти вырезы к узкополосным фильтрам отношения не имеют
впрочем советую тебе пойти и речь под поезд как можно быстрей.
Аноним 14/11/16 Пнд 06:12:19 #236 №396325 
>>396309
>откуда ты узенькие полоски взял? откуда твою мать ты их высрал?
ИДИ ПОД СТРУЮ СУКА МОЙСЯ

ИДИ СУКА В ГУГЛ ЗАГУГЛИ
ВОЗЬМИ ЗАДЕЙСТВУЙ СВОИ КУЛЬТИ НАЖМИ НА КЛАВИШЫ
НАЖИМАЙ ВОТ ТАК
У З К И Й С П Е К Т Р П О Г Л О Щ Е Н И Я
забиваешь это в переводчик (ведь англ ты не знаешь), потом оттуда берёшь так КОПИРОВАТЬ ВСТАВИТЬ и КАРТИНКИ и ищи, сука мразь, ищи там в картинках спектры какой-нибудь мандавошной органики с узкими пиками точёными а не хуями дрочеными на которых ты восседаешь от рождения понял сука урод говно.
Аноним 14/11/16 Пнд 06:15:14 #237 №396326 
11303.jpg
>>396317
пикрелейтед моя реакция на Архипиздрического волшебного урода-дебила.
>>396320
не отвлекайся на одноклеточных петушара продолжай сосать мой хуй
мы науку блять делаем.
Аноним 14/11/16 Пнд 06:25:26 #238 №396327 
>>396320
о таких фильтрах в оптике не слышал, но не суть, буду знать. А до этого говорили об этом:
http://golddragonoptics.ru/uzkopolosnye-filtry.html
>>396322
>>396323
>>396324 - можно изменить характеристику отражающей поверхности, вместо источника, но ты тупой еблан и думать не умеешь.
>>396325
Хуя ты подрвался, а теперь совсем вывернись наизнанку, напомиаю как звучал вопрос:
>Можно ли создать краску, которая на солнце белая, а в белом светодиодном освещении черная?
Аноним 14/11/16 Пнд 06:32:41 #239 №396328 
>>396309
>А теперь говноед, пофантазируй, с узкополосными фильтрами, если у тебя один единственный светодиод.

хули фантазировать
фантазируешь ты когда лезешь в штаны но находишь там второе анальное отверстие вместо члена
или когда пишешь 10 лет опыта физика-пиздунишки


изи катка для школьника 8го класса интерната для говорящих жопных полипо:
сфокусировал светодиод белый на призму
разложил в спектр призмой особо задроченной на три луча получил ТОТ ЖЕ СУКА ТРЕХКОМПОНеНТНИК
дальше идёшь в клуб в москве для пидоров
начинаешь сосать у всех подряд
насасываешь килопарсек грязных хуев по 10р/отсос
копишь на морозильник и генератор и бассейн соляры едешь с этим дерьмом в тайгу
замораживаешь себя нахуй
ждешь
тебя пидоры-активисты-копчёные типа докинза размораживают как ветерана спермоглота в будущем через 100 лет
ты сосёшь еще пару лет и покупаешь себе на рынке у цыган мозг кибермандавошки Mark 1337
берешь отвертку суешь в ухо до упора
достаешь обратно наклоняешься ждёшь пока из черепа не вытечет тухлая сперма заменявшая тебе мозг
вставляешь мозг с рынка начинаешь соображать худо бедно (но лучше чем божественный чудаун выше) и узнаешь что такое спектры и как с ними работать как физик
вспоминаешь что я тут выше писал и получаешь решение.
светишь на краску диодом - чёрная - пускаешь слюни
светишь солнцем - белая - получаешь инсульт от волнения протягиваешь копыта и догниваешь тихо в канаве



Аноним 14/11/16 Пнд 06:38:37 #240 №396329 
>>396327
хули ты доебался еблан
я тебе уже два решения дал готовых инновационных с любыми диодами

хочешь быть примитивным чмом таджиком без души - бери делай сдвиг люминофором
Аноним 14/11/16 Пнд 09:01:10 #241 №396333 
>>396329
укажи свои посты, если ты не этот >>396328 мудель.
Аноним 14/11/16 Пнд 09:30:47 #242 №396337 
>>396257
бамп
Аноним 14/11/16 Пнд 12:04:01 #243 №396346 
Правда ли, что для создания активных источников холода и темноты нужна отрицательная энергия и никак иначе?
Аноним 15/11/16 Втр 01:37:00 #244 №396389 
>>396346
Ученые смогли создать лучи темноты без всяких отрицательных энергий, гугли.
Аноним 15/11/16 Втр 11:44:27 #245 №396408 
>>396346
вон холодильник у тебя дома использует отрицательную энергию для заморозки продуктов?
Аноним 15/11/16 Втр 14:28:44 #246 №396426 
>>396257
Ибо при остывании в нем образуются микрополости (много мелких пузырьков).
Аноним 15/11/16 Втр 18:01:17 #247 №396469 
Водяной лёд твёрдый из-за водородных связей?
Почему при нуле происходит кристализация - резкий фазовый переход?
Выходит с понижением температуры крепкость водородных связей растёт?
Аноним 15/11/16 Втр 19:39:00 #248 №396476 
>>396469
>крепкость водородных связей растёт
Нет. Молекулы дрыгаются сильнее.
Аноним 16/11/16 Срд 18:29:47 #249 №396548 
inner.jpg
поясните как происходит взаимодействие электрона с ядром. электрон с ядром обменивается фотонами. почему фотоны летают м-ду электроном и ядром, а не улетают в ебеня? с какой частотой происходит обмен фотонами?
Аноним 16/11/16 Срд 23:53:07 #250 №396565 
main-qimg-b8eab5cb0a93266270d020f89fbe311e-c.jpg
>>396548
ядро создаёт стоячую волну, из которой электрон может выбраться лишь с хорошим пинком (и то скорей всего застрянет в следующей колее)
%%наглядный, хоть и грубый пример можешь получить следующим экспериментом: включи фен и установи его над бутылкой таким образом, чтоб она начала гудеть - если комната достаточно большая - ты заметишь, что громкость звука изменяется волнообразно (не знаю работает ли с бутылкой - я с тыквой-горлянкой этот эффект наблюдал - впрочем, не исключено, что я наблюдал резонанс звука с отражённым от стен)
Аноним 17/11/16 Чтв 03:23:51 #251 №396568 
>>396565
Ой, не трали школьников.
Аноним 17/11/16 Чтв 16:18:58 #252 №396591 
Внимание, вероятно платиновый вопрос!
Объясните процессы по данному примеру:
Вокруг Земли построили железную дорогу и гипотетический поезд едет со скоростью 99% от скорости света. Поезд проезжает вокруг Земли шесть раз (примерно 1 секунда, насколько я помню).
1. Что будет происходить внутри поезда?
2. Что будет происходить снаружи?
3. Что увидит пассажир, взглянув в окно?
4. Что увидит человек вне поезда?
Аноним 17/11/16 Чтв 16:41:21 #253 №396595 
>>396591

улетит поезд в космос
Аноним 17/11/16 Чтв 16:52:14 #254 №396597 
>>396595
Действительно.
Аноним 17/11/16 Чтв 17:18:37 #255 №396599 
>>396591
1. ничего особого (время так замедляется тип)
2. ну быстра так все (ну время тип относительно ускоряется)
3. нифига не понятно, слишком быстро
4. ну быстро едет ну и чо?
все основано на теории относительности
Аноним 17/11/16 Чтв 17:25:59 #256 №396600 
>>396591
Поезд жестоко распидорасит об воздух, а его остатки сгорят от трения с атмосферой. Рельсы распидорасит, все распидорасит. Он не сможет проехать на такой скорости и нанометра, моментально наступит взрыв и пиздец. Ни о каких кругах речи идти не может.
1) Первые пикосекунды пиздец, а потом сам поезд перестанет существовать, он превратится в пар
2) Пожары, летащие на гигантской скорости обломки, большой кратер
3) Человек не успеет посмотреть в окно, все распидорасит за пикосекунды если не быстрее
4) Пожары, взрывы от столкновений на больших скоростях, пиздец, кратер
Аноним 17/11/16 Чтв 21:35:03 #257 №396607 
>>391093
>Гравитация либо мгновенна
>Ничего, она и так непостоянна. В вакууме одна, в жидкости другая

Зачем отвечать, если ты тупой идиот?
Аноним 17/11/16 Чтв 22:59:24 #258 №396613 
>>396591
А если лом кинуть в этой ситуации в унитаз?
Аноним 18/11/16 Птн 09:54:36 #259 №396628 
Стикер
>>396607
потому что не тупые идиоты в саентаче вопросы не задают. они их в гугле задают
Аноним 18/11/16 Птн 15:42:09 #260 №396646 
>>390896 (OP)
Аноны, а что произойдет если один из запутанных фотонов отправить а черную дыру, за горизонт событий?
Аноним 18/11/16 Птн 15:49:23 #261 №396650 
>>396646
ты не поверишь. НИЧЕГО
Аноним 18/11/16 Птн 18:04:54 #262 №396658 
>>390896 (OP)
http://www.nature.com/news/stephen-hawking-there-are-no-black-holes-1.14583
OTO sosnooley?
Аноним 19/11/16 Суб 04:10:30 #263 №396690 
1. за счёт чего увеличивается масса тела при движении со скоростью ? ведь количество молекул и атомов в нём не меняется кардинально. Откуда появляется масса ?
2. Что произойдёт внутри стальной коробке с толщиной стен в несколько метров(или километров, если хотите) если в ней взорвать гранату ? просто повысится давление и температура или коробку разорвёт не смотря на толщину стен ? допустим, что полость в этой коробке не многим больше размеров гранаты, ну например 20х20 см.
Аноним 19/11/16 Суб 07:39:35 #264 №396696 
>>396690
> за счёт чего увеличивается масса тела при движении со скоростью ?
За счет более трудного воздействия на быструю частицу измеряющего магнитного поля. То есть увеличение массы в бесконечность - кажущееся явление, при котором реальная масса (которая инерционная) не растет.
Аноним 19/11/16 Суб 07:46:14 #265 №396697 
Если в мозге сигналы передаются при помощи электрического заряда, то откуда он тогда берется изначально? Как в мозге может появиться заряд?
Аноним 19/11/16 Суб 15:33:02 #266 №396747 
>>396697
Как и в аккумуляторах - химия.
Аноним 19/11/16 Суб 17:49:08 #267 №396765 
>>396747
Он хранится при помощи химии, а откуда он туда попадает?
Аноним 19/11/16 Суб 19:02:32 #268 №396767 
>>396765
Ты поел, продукты переварились и выработалась энергия - часть этой энергии, а именно 20% забирает мозг, трансформирую её в электричество. так то так
Аноним 19/11/16 Суб 19:19:35 #269 №396769 
>>396765
Вот это почитай Stephen H. Wright, Advan. Physiol. Edu. 28: 139-142, 2004.
Аноним 19/11/16 Суб 20:13:01 #270 №396775 
>>396767
тепловая энергия?
Аноним 19/11/16 Суб 20:23:11 #271 №396779 
>>396775
>>396769
Аноним 19/11/16 Суб 20:57:07 #272 №396783 
Помогите, совсем запутался! Взаимодействуют ли как-то индукторы магнитного поля между собой посредством этого поля, или все происходит лишь за счет сил Лоренца?
Пример: Два магнита отталкиваются противоположными полюсами, это все знают, но вот по какой именно причине они это делают? В силу закона Лоренца, воздействуя на заряды противоположного магнита, или же реально каким-то чудесным образом посредством магнитного поля?
Второй пример: На вектор магнитной индукции постоянной длины надета трубка. Снаружи трубки магнитного поля нет. Вокруг трубки принудительно вращается электрон.
Вопрос: Будет ли на них действовать какая-то сила, связанная с тем самым вектором в трубке?
Аноним 19/11/16 Суб 20:58:39 #273 №396784 
>>396783
Будет ли на *него
Аноним 19/11/16 Суб 21:02:11 #274 №396785 
>>396783
В силу закона Лоренца, воздействуя на заряды противоположного магнита посредством магнитного поля
Аноним 19/11/16 Суб 21:03:34 #275 №396786 
>>396783
>На вектор магнитной индукции постоянной длины надета трубка. Снаружи трубки магнитного поля нет.
ЧОООООООООООООООООООО?
Аноним 19/11/16 Суб 21:07:19 #276 №396787 
>>396786
надел тобi трубку
Аноним 19/11/16 Суб 21:13:46 #277 №396788 
>>396786
Ну внутри поле есть, вдоль трубки, снаружи нет, что непонятного?

>>396785
То есть второй пример - ничего не будет действовать?
Аноним 19/11/16 Суб 21:22:09 #278 №396789 
>>396788
Как на вектор надеть трубку?
Аноним 19/11/16 Суб 21:30:55 #279 №396790 
>>396789
С восторгом.
А если серьезно, отключаем магнитное поле, располагаем трубку на одной оси с будущим вектором магнитной индукции и включаем магнитное поле. Пример конечно сферический в вакууме, как и все примеры в учебниках.
Аноним 19/11/16 Суб 21:32:18 #280 №396791 
Ну скажите уже, будет ли во втором примере действовать или нет?
Аноним 19/11/16 Суб 21:36:28 #281 №396792 
>>396791
Блэт, ты заебал, у тебя плотность энергии выражается через квадрат индукции, например, потому, в любом случае, если поле не постоянное, будет энегрия не равная энегриям отдельно поля от электрона и поля внутри трубки, если у тебя граничных условий на поле электрона нет, разумеется. Ты это хотел услышать, или что-то другое?
А, вообще, у меня на районе за такие вопросы убивают нахуй.
Аноним 19/11/16 Суб 21:37:25 #282 №396793 
>>396791
Допустим есть постоянный магнит. Чтобы убрать его поле, придется его засунуть внутрь сверхпроводника. Этот магнит не будет действовать на тела вне сверхпроводника.
Аноним 19/11/16 Суб 22:07:34 #283 №396794 
>>396792
Я же написал что поле постоянное. И я так и не понял, будет воздействовать это постоянное поле с вихревым полем электрона или нет?
Аноним 19/11/16 Суб 22:22:02 #284 №396796 
>>396794
Если у тебя стенка "непроницаемая" только для постоянного поля внутри, будет, если нет, то нет, как ни странно.
Аноним 19/11/16 Суб 22:33:36 #285 №396797 
>>396796
Все верно, она непроницаема только для постоянного внутри. Точнее оно и не пытается проникнуть, т.к. линии магнитного поля вдоль стенок расположены, но не суть. Пример как я уже говорил, сферический.
Но почему тогда будет? Где здесь воздействие на заряд со стороны поля?
Аноним 19/11/16 Суб 23:50:30 #286 №396811 
>>396797
Это взаимодействие полей без заряда, распиши тензор энергии импульса твоей ебанутой хуеты, и обнаружь, внезапно, свои произведения компонентов полей, которые для какого-то ебанутого контура, по которому движется заряд, будут давать потоки энергии и импульса.
Аноним 20/11/16 Вск 13:12:20 #287 №396846 
14796340273090.jpg
В чём подводные камни?
Аноним 20/11/16 Вск 13:18:21 #288 №396850 
>>396793
нивчем
Аноним 20/11/16 Вск 17:10:40 #289 №396876 
123455.png
Добрый вечер,
Вопрос уровня 11го класса общеобразовательной школы, но гугл не помог.
Как в данной ситуации искать traction force (подчеркнуто красным)? Правильно ли я понимаю что это то же самое что tractive force? Определение я кажется нашел, но вывести из него какую то формулу не могу и она нигде не дана. Прочитал уже почти все главы по механике в учебнике, но ничего похожего там тоже не нашел.
Аноним 20/11/16 Вск 17:20:32 #290 №396878 
>>396846
Исключительно в крайней хуевости оптических телескопов на таких дистанциях. От Солнца до Земли 8 минут, если даже телескоп будет передавать цифровую картинку твоей жопы прямо из космоса, вместо зеркала (что более практично), представь какое расстояние нужно хотя бы на час.
Аноним 20/11/16 Вск 17:23:24 #291 №396880 
>>396878
...на таком расстоянии фотоны, которые отразились от твоей жопы и полетели к телескопу будут испытывать достаточные гравитационные, ЭМ и другие воздействия во время полета так, что конечная картинка в телескопе превратится в размазанную хуйню.
Аноним 20/11/16 Вск 17:25:44 #292 №396881 
>>396811
Получается, получаем импульс без противодействия?
Аноним 20/11/16 Вск 19:52:03 #293 №396897 
>>396775
Химическая
Аноним 20/11/16 Вск 20:25:13 #294 №396902 
>>396846

в том надо долететь и построить быстрее чем ты умрешь
Аноним 20/11/16 Вск 23:08:38 #295 №396919 
Почему ночью облаков не видно? Я звезды вижу, хотя днем облака могут закрывать солнце.
Аноним 21/11/16 Пнд 09:37:48 #296 №396942 
>>396919
Почему не видно? У меня а городие видно.
Аноним 21/11/16 Пнд 11:45:15 #297 №396956 
>>396942
Хуй знает. Ночью только звезды вижу. Но если смотреть из космоса - облака есть и их дохуя.
Аноним 21/11/16 Пнд 12:21:34 #298 №396960 
>>396956
Как часто бываешь в космосе?
Аноним 21/11/16 Пнд 12:29:32 #299 №396963 
>>396960
Вчера был, например.
https://www.youtube.com/watch?v=UGPuEDyAsU8
Аноним 21/11/16 Пнд 12:34:19 #300 №396964 

>>396846
В том, что диаметр телескопа больше солнечной системы, возможно, в разы или на порядки.
Аноним 21/11/16 Пнд 13:55:29 #301 №396974 
>>396646
Он поглотится. Ваш КО.
Аноним 21/11/16 Пнд 20:13:45 #302 №397015 
800px-Windchilleffecten.svg[1].png
Как считать "погоду по ощущениям".

Знаю есть Heat Index - юзается при температуре выше около 20, и с вводом влажности. Есть ветро-холодовой индекс, юзается когда меньше 10 градусов и с вводом скорости ветра.

Как считать ощущения между?

На америкоском сайте нац. погодных прогнозов нарыл что - пофигу, берется просто текущая температура. Но, допустим, в погодной приложухе AccuWeather - данные по текущей и ощущаемой температуе отличаются для 10 - 20 градусных величин. И есть еще формула для apparent temperature - он вроде считает и по влажности и по скорости ветра. Но чот хз норм ли.

Пик рандом.
Аноним 22/11/16 Втр 00:40:54 #303 №397060 
Посоветуйте что по статмеханике, чтобы я смог перейти как человек к solid state physics.
Аноним 22/11/16 Втр 15:50:12 #304 №397108 
>>396876

traction force это сила тяги. скорее всего надо массу автомобиля умножить на его ускорение на этом участке. ускорение это скорость изменения скорости. или скорость в точке Б минус скорость в точке А делить на время, прошедшее между точками А и Б
Аноним 22/11/16 Втр 19:39:26 #305 №397126 
ИТАК
Какова вероятность бросая 2 шестигранных кубика выбросить дубль 5 раз подряд?
Аноним 22/11/16 Втр 22:08:32 #306 №397138 
>>397126
1/6^10.
Аноним 22/11/16 Втр 22:46:54 #307 №397139 
>>397138
Неверно.
Аноним 22/11/16 Втр 22:55:02 #308 №397140 
>>397126
(6*((1/6)^2))^5
Аноним 22/11/16 Втр 23:25:35 #309 №397142 
>>397108
Благодарю
Аноним 23/11/16 Срд 07:09:15 #310 №397154 
>>396974
Второй сопьется и однажды морозной ночью поглотится скалярным полем.
Аноним 23/11/16 Срд 07:10:47 #311 №397155 
>>397126
тебя рулетка наебала?
Аноним 23/11/16 Срд 07:13:37 #312 №397156 
>>396646
Энштейн тебе луч откусит. А если серьезно, где ты будешь хранить второй фотон, пока первый будешь запихивать в черную дыру? А представь каково ему.
Коробок фотонов.jpg
Аноним 23/11/16 Срд 07:16:37 #313 №397157 
>>397156
А если еще серьезней, то можна выпустить их оба, один в ЧД, другой немного вбок, где на расстоянии как до ЧД будет ждать ученый с прибором и смотреть что будет с фотоном. Только вот как весь полет поддерживать спутанность - не ясно. Это даже теоретически хрен себе представишь, даже Поппером запахло.
Аноним 23/11/16 Срд 12:55:54 #314 №397170 
3D2.jpg
Наконец-то тредж, который я искал.

Итак, вопрос следующий. Если в космосе у космического корабля двигатели работают, скажем, бесконечное количество времени, будет ли корабль бесконечно разгоняться, или лишь до определенной скорости? Там же нет трения воздуха. Всякими космическими телами пренебречь.
Аноним 23/11/16 Срд 13:30:41 #315 №397173 
>>397170
Его скорость будет асимптотически приближаться к скорости света в вакууме, но никогда её не превысит.
Аноним 23/11/16 Срд 13:40:56 #316 №397176 
>>397173
Что-то я не упоминул об этом. В общем, насрать на скорость света, главное, что оно будет постоянно разгоняться на одной и той же тяге. Да?
Аноним 23/11/16 Срд 13:57:02 #317 №397179 
>>397170
>или лишь до определенной скорости?
Да, эта скорость зависит от мощности двигателя и локальной вязкости вакуума, может быть как меньше скорости света, так и больше.
Аноним 23/11/16 Срд 14:06:20 #318 №397181 
>>397173
Если двиг по типу emdrive то так, но если двиг гравитационный, то ему насрать на скорость света.
Аноним 23/11/16 Срд 14:39:35 #319 №397183 
>>397181
Я гуманитарий, можно по-проще? Из космических двигателей знаю только что такое реактивный.
Аноним 23/11/16 Срд 14:42:37 #320 №397184 
>>397176
Нет. Чем ближе скорость с скорости света, тем меньше ускорение будет. Это тебе не механика Ньютона, масса корабля будет постоянно расти, вследствие чего по a=F/m ускорение будет падать.

Чтобы тело, обладающее массой, разогнать до скорости света - нужно бесконечную энергию затратить.
Аноним 23/11/16 Срд 14:43:17 #321 №397185 
>>397181
>>397183
Два дебила - это сила.
Аноним 23/11/16 Срд 14:43:18 #322 №397186 
>>397183
Это фрик пишет. Не существует такой штуки, как "гравитационный двигатель". Ну и по современным данным скорость распространения гравитации равна скорости света в вакууме.
Аноним 23/11/16 Срд 14:43:24 #323 №397187 
>>397139
А черт, твоя правда, я почему-то представил дубль как две шестерки, а у двух кубиков 6 возможных дублей, этот >>397140 прав.
Аноним 23/11/16 Срд 14:46:32 #324 №397189 
>>397176

Так-то да, будет постояно разгоняться, другое дело что ускорение будет становиться всё меньше.

Наверни Википедии:
Согласно специальной теории относительности, энергия объекта с массой покоя m и скоростью v равна γmc2, где γ — определенный выше фактор Лоренца. Когда v равна нулю, γ равен единице, что приводит к известной формуле эквивалентности массы и энергии E = mc2. Поскольку фактор γ приближается к бесконечности с приближением v к c, ускорение массивного объекта до скорости света потребует бесконечной энергии. Скорость света — это верхний предел скорости для объектов с массой покоя. Это экспериментально установлено во многих тестах релятивистской энергии и импульса[27].

https://ru.wikipedia.org/wiki/%D0%A1%D0%BA%D0%BE%D1%80%D0%BE%D1%81%D1%82%D1%8C_%D1%81%D0%B2%D0%B5%D1%82%D0%B0
Аноним 23/11/16 Срд 14:49:28 #325 №397190 
>>397170
С точки зрения внешнего наблюдателя ускорение корабля будет падать (время корабля замедляется по сравнению с временем наблюдателя), а скорость приближаться к скорости света.
С точки зрения экипажа ускорение корабля будет постоянным, а вселенная вокруг будет изменяться с ебической скоростью, вплоть до того, что за секунду на корабле снаружи будут проходить миллиарды лет (положено в основу сюжета "Тау Ноль" Андерсона).
Аноним 23/11/16 Срд 14:49:42 #326 №397191 
>>397186
>>397189
Отстань.
Аноним 23/11/16 Срд 14:50:51 #327 №397192 
>>397190
Кстати да, двачну тебе. Вопрошающий не указал системы отсчёта наблюдателя.
Аноним 23/11/16 Срд 14:51:23 #328 №397193 
>>397190
>>397192
ето все жидовские выдумки
Аноним 23/11/16 Срд 16:45:38 #329 №397200 
>>397190
Ебать, а время-то каким хуем так изменится?
Аноним 23/11/16 Срд 16:57:27 #330 №397201 
>>397200
>>397193
Хуем ЖыдоЭйнштейна оно так изменится.
https://ru.wikipedia.org/wiki/Эксперимент_Хафеле_—_Китинга
Аноним 23/11/16 Срд 17:01:07 #331 №397203 
>>397200
Как это не странно, на люрке есть статья, хорошо поясняющая эти эффекты: http://lurkmo.re/Теория_Относительности
Аноним 23/11/16 Срд 17:01:11 #332 №397204 
>>397183
Реактивный движитель это слишком неэффективно, только отсталые в развитии на таких летают. На него топлива не напасешься. Гораздо эффективнее не таскать среду для отталкивания с собой, а использовать ту что находится вокруг в пространстве. На этом принципе основан emdrive, он словно пропеллер отталкивающийся от воздуха, только отталкивается не от воздуха, а от вакуума - там тоже есть весомая среда, на которую можно опираться. Но у эмдвигла есть ограничение по скорости, поскольку он использует в качестве движущей силы фотоны, то не может летать быстрее фотонов. Если юзать вместо фотонов гравитацию, то можно беспрепятственно разгоняться выше световых скоростей, поскольку гравитация на много порядков быстрее.
Аноним 23/11/16 Срд 17:06:36 #333 №397205 
>>397204
Уймись, фрик ебучий. Скорость распространения гравитационной волны равна скорости света в вакууме.

А теперь покинь раздел и направляйся в /sf/.
Аноним 23/11/16 Срд 17:09:44 #334 №397206 
>>397203
>Теория_Относительности
Именно эта теория виновна в торможении науки. Сейчас бы уже на Альдебаран летали, если бы не она. Не рекомендую изучать отключив критическое мышление и не ознакомившись с состоянием науки до нее, иначе уверуете и будете как верующие верить в сказки. Да и вообще изучать ее, кроме как в исторических целях, бесполезно, только время зря потратите.
Аноним 23/11/16 Срд 17:12:33 #335 №397207 
>>397205
>Скорость распространения гравитационной волны равна скорости света в вакууме.
Пруфов нет.
Аноним 23/11/16 Срд 17:29:45 #336 №397209 
>>397206
Выкинь свой GPS-навигатор, пожалуйста. GPS не работает без ТО.
Аноним 23/11/16 Срд 17:30:19 #337 №397211 
>>397206
Что в ней не верно? Т. о. не противоречит ни движению быстрее скорости света, ни путешествию во времени. Вселенная не обязана тебе нравиться.
Аноним 23/11/16 Срд 17:30:40 #338 №397212 
>>397207
O RLY?

11 февраля 2016 года было объявлено об экспериментальном открытии гравитационных волн коллаборациями LIGO и VIRGO[13][14][15]. Анализ влияния события GW150914 на дисперсию гравитационных волн в зависимости от частоты не противоречит гипотезе о нулевой массе гравитона и совпадении его скорости со скоростью света для гипотетических расширений ОТО (оценка сверху на массу гравитона: mg ≤ 1.2 × 10−22 eV/c2, соответствует оценке снизу на скорость для частоты 35 Гц: vg/c ≤ 1 - 10-18)[16]
Аноним 23/11/16 Срд 17:32:11 #339 №397213 
>>397211
Эфироребёнок же.

Что забавно - если-бы эфирные теории были верны - то точно не летали-бы быстро. Т.к. взаимодействие с эфиром, электроны выдувает из атомов, гроб, кладбище, пидор.
Аноним 23/11/16 Срд 17:36:15 #340 №397214 
Вопрос - могут ли существовать антифотоны, частицы с нулевой массой и отрицательным запасом энергии? Я знаю, что таких не открыли, но всякие тахионы то вроде как не опровергли.
Аноним 23/11/16 Срд 17:38:09 #341 №397215 
>>397212
Все это вранье и выдумки.
>>397211
>Что в ней не верно?
Всё.
Аноним 23/11/16 Срд 17:38:19 #342 №397216 
>>397214
Сейчас считается, что фотон и антифотон - это то же самое.
Аноним 23/11/16 Срд 17:41:58 #343 №397217 
>>397215
О, вот и ВРЁТИ.

Факты не укладываются в твой пропахший эфиром мирок - отрицаем, ясно, понятно.

Майкельсон и Морли тоже нашли эфирный ветер, да, но ЗОГ скрывает?
Аноним 23/11/16 Срд 17:44:01 #344 №397219 
>>397217
>Факты
Это не факты, пруфоф нет.
>но ЗОГ скрывает?
Вот видишь, ты сам все понимаешь.
Аноним 23/11/16 Срд 17:47:53 #345 №397220 
>>397219
Если данные от LIGO для тебя не пруф, то что тогда пруф?

Кстати, зачем ЗОГ скрывает?
Аноним 23/11/16 Срд 17:51:08 #346 №397221 
>>397216
В классическом понимании слова "античастица" да, но я имею ввиду именно безмассовую частицу с отрицательным запасом энергии. У фотона же есть запас энергии, так?
Аноним 23/11/16 Срд 17:59:02 #347 №397223 
>>397219
Даже если ТО не верна, для разгона двух киллограмовой железки понадобится энергия, эквивалентная аннигиляции киллограма вещества. Это 90 000 000 000 000 Дж, или 90 триллионов Дж. Это 21 мегатонна тротилового эквивалента.
Аноним 23/11/16 Срд 17:59:17 #348 №397224 
>>397220
>LIGO
Это не наука а сплошные распилы и обман.
Давай ссылку на .рт с пруфами на наши исследования.
>зачем ЗОГ скрывает?
Потому что жиды.
Аноним 23/11/16 Срд 17:59:53 #349 №397225 
>>397223
>Это 21 мегатонна тротилового эквивалента.
Совсем немного для эфирной энергии.
Аноним 23/11/16 Срд 18:09:50 #350 №397226 
>>397225
Тэксь. Эфирная энергия - это простите что?
Аноним 23/11/16 Срд 18:12:51 #351 №397227 
>>397226
Это энергия эфира. Ты глупый, да?
Аноним 23/11/16 Срд 18:21:32 #352 №397228 
>>397227
Тогда тепло - энергия камней. Светоносный эфир, на сколько я помню, среда для распространения света, относительно которой и можно рассчитать скорость света. Никаких других свойств ему не приписывали, в том числе и возможность добыть из него дохрена бесплатной энергии. Так что либо ты мне расписываешь, что такое эфирная энергия и как её можно преобразовать в кинетическую, либо идешь нахуй со всеми своими устаревшими теориями XIX века.
Аноним 23/11/16 Срд 18:30:10 #353 №397229 
>>397228
>возможность добыть из него дохрена бесплатной энергии
Да, ето так.
>что такое эфирная энергия и как её можно преобразовать в кинетическую
Научись пользоваться гуглом, ето совсем не больно.
Аноним 23/11/16 Срд 18:39:18 #354 №397230 
>>397221
Вот честно говоря - не представляю себе, что есть отрицательный запас энергии. Извини, тут моих знаний не хватит (я не физик).
Аноним 23/11/16 Срд 18:40:28 #355 №397231 
>>397229
Гугл дает ссылки вроде этой:
https://professionali.ru/Soobschestva/otkroveniya_lyudyam_novogo_veka/energii/
Или ссылки, где говорят, что трансформаторы, которые на ЖД активно используются в системах СЦБ и энергию не вырабатывают (сам с ампер- и вольтметром мерил), выдает 100 кватт. Ты о каких говоришь?
Аноним 23/11/16 Срд 19:01:46 #356 №397237 
>>397231
Очень плохо гуглишь.
Полон интернет рабочих схем, даже на русском языке.
Вас вконтакте или одноклассниках или откуда ты пришел гуглить не учат чтоли?
Аноним 23/11/16 Срд 19:05:07 #357 №397239 
>>397237
Так-так, и что же это за рабочие схемы? Т.н. "качер"? Всякие якобы работающие схемы (работающие за счёт того, что простые измерительные приборы не регистрируют высокочастотные сигналы/реактивную мощность?)

Радиолюбитель-кун достал банку хлорного железа и готов облить эфирного зомби.
Аноним 23/11/16 Срд 19:07:52 #358 №397240 
>>397239
>измерительные приборы не регистрируют
>врети ваши приборы не приборы
Ну тут все ясно.
Аноним 23/11/16 Срд 19:11:51 #359 №397244 
>>397240

О чём ты? Бытовой счётчик не регистрирует реактивную мощность. Авометр (тестер) за тысячу рублей начинает страшно лажать, если в измеряемом токе есть высокочастотные пульсации.

Возьми осциллограф с достаточно широкой полосой пропускания - и всё видно будет.
Аноним 23/11/16 Срд 19:29:57 #360 №397248 
>>397244
Не оправдывайся, "специалист".
Приборы у него не приборы, пруфы не пруфы, энергия не энергия.
Вот из-за тебя наука и в тупике.
Аноним 23/11/16 Срд 19:33:55 #361 №397249 
>>397248
Это у тебя LIGO с чего-то не пруф.

В общем - я думаю, что ты толсто троллишь, и не хочу больше тратить на тебя время, будь ты хоть троллем, хоть настоящим эфирным зомби.

Иди, строй свои сверхъединичные генераторы и возвращайся, когда разбогатеешь на них.
Аноним 23/11/16 Срд 20:22:03 #362 №397253 
>>397237
Полон рабочих схем? Мне не нужны схемы, я их собирать не собираюсь. А вот описание принципов с ссылками на эксперименты пригодится. Есть такое? Предположим, в гугле меня забанили.
Аноним 23/11/16 Срд 20:42:32 #363 №397255 
>>397249
>я думаю, что ты толсто троллишь
Но как Вы столь стремительно догадались, Холмс? :3
>>397253
>Предположим
Услуги оператора гугла - 10€ за запрос.
Аноним 23/11/16 Срд 20:54:28 #364 №397257 
Возможно ли существование галактики из антиматерии?
Аноним 23/11/16 Срд 20:55:26 #365 №397258 
>>397257
Да нет.
Аноним 23/11/16 Срд 21:03:08 #366 №397260 
>>397255
Эх, жалко. Я надеялся встретить настоящего зоговца, поболтать, выслушать его бредни, опровергнуть их... Это весело, знаешь ли. Не обламывай так больше, ок?
Да, и еще ты сильно палишься. Слушай и записывай:
Тру зоговец не оскорбляет ("глупый да?" считается).
Тру зоговец наизусть знает все принципы работы всяких вечных двигателей.
Тру зоговец точно знает кто, зачем, почему и что скрывает.
Тру зоговец обязательно будет ссылаться на какого либо лжеученого, или какой либо сайт.
Я в принципе тебя подозревал, просто верил в желаемое, благо на неадекватных у меня не горит. Но в принципе есть потенциал, ты довольно хорош. Но лучше пройти курсы повышения квалификации, побольше пообщаться с настоящими придурками, отрицающими современную науку.
Аноним 23/11/16 Срд 21:04:12 #367 №397262 
>>397257
Да. Путешествовать в неё не советую, черевато ядерными взрывами.
Аноним 23/11/16 Срд 21:10:06 #368 №397263 
>>397260
>Тру зоговец не оскорбляет
Мальчик, сколько тебе лет такому наивному?
>зоговец наизусть знает
>зоговец
>знает
Совсем мало лет.
>зоговец обязательно будет ссылаться
Единственно более-менее верно.
>просто верил
-> /re
Аноним 23/11/16 Срд 21:16:44 #369 №397264 
>>397262
Прямо таки взрывами? Я думал, что как попроще аннигиляция она же? будет, поспокойнее.
Аноним 23/11/16 Срд 21:22:57 #370 №397265 
>>397264
Да, аннигиляция.
Представь сколько энергии получится из 100кг тебя и 100кг антитебя, и сравни с средней атомной бомбой
Аноним 23/11/16 Срд 21:27:54 #371 №397266 
>>397265
>сколько энергии получится из 100кг тебя и 100кг антитебя
Совсем немного по сравнению с эфирной энергией
Аноним 23/11/16 Срд 21:42:58 #372 №397269 
>>397263
Я с этими идиотами в перепалках не один день провел. Хотя может имел дело с конкретным вежливым подвидом. Учитывая, что они были руско-нациками, их вежливость не делает им особенной чести. Но подумай, у такого нет намерения оскорбить - только обратить в свою секту и поругать ученых.
И я не про тех, кто в /re сидят, там ПГМ-ные, я про тех, кто сидит в /zog
Аноним 23/11/16 Срд 21:46:15 #373 №397270 
>>397264
Как уже писал выше, килограмм аннигилрованного вещества - эквивалент 21-мегатонного взрыва. На самом деле меньше, 10 мегатонной, половина энергии в нейтрино уходит, но тебе хватит.
Аноним 23/11/16 Срд 21:50:55 #374 №397271 
>>397269
>Я с этими идиотами в перепалках не один день провел.
Подобное к подобному.
Аноним 23/11/16 Срд 21:56:26 #375 №397272 
>>397271
Фи. Как грубо.
Аноним 23/11/16 Срд 21:59:09 #376 №397273 
>>397272
>Как грубо.
Ну не обижайся, няша, ведь бьет - значит любит :3
Аноним 24/11/16 Чтв 01:02:08 #377 №397285 
>>397257
И самый прикол в том что они(антиматерия это глактики) излучают те же самые фотоны, той же самой длины волны. Т.е. визуально ты не сможешь отличить воду от антиводы, хлор(зеленоватый газ) от антихлора(тоже будет зеленым), фтор(желтый газ) от антифтора(тоже будет желтым) и т.д.
И самый прикол в том что астрофизик по телескопам нихуя не сможет различить что в эта галактика состоит из материи а вот эта из антиматерии.
Почему же тогда считается что все галактики состоят из материи? Потому что из-за слабого взаимодействия и нарушения какой-там симметрии антиматерия долго не живет и распадется.
Аноним 24/11/16 Чтв 11:46:19 #378 №397320 
>>397285
А может есть параллельная вселенная, где распадается материя?
Аноним 24/11/16 Чтв 11:53:57 #379 №397321 
>>397189
>Скорость света — это верхний предел скорости для объектов с массой покоя
>массой покоя
Ну мы же не массивный гравитон рассматриваем.
Аноним 24/11/16 Чтв 11:55:04 #380 №397322 
>>391308
Чертов гений
Аноним 24/11/16 Чтв 11:56:27 #381 №397323 
>>397209
> GPS не работает без ТО
На Хабре об этом прочитал, умненький?
Аноним 24/11/16 Чтв 11:58:59 #382 №397324 
>>397221
Прочитай про вирт. частицы.
Аноним 24/11/16 Чтв 12:32:38 #383 №397331 
>>397321
А космический корабль, про который речь шла у нас имеет нулевую массу покоя, да?
Аноним 24/11/16 Чтв 12:33:40 #384 №397332 
>>397323
Нет.
Аноним 24/11/16 Чтв 12:35:35 #385 №397333 
>>397285
Считается потому, что нигде не видно аннигиляции. Если-бы где-то в обозримом космосе были объекты из антиматерии - мы-бы видели, как они-бы аннигилировали.

>>Потому что из-за слабого взаимодействия и нарушения какой-там симметрии антиматерия долго не живет и распадется.

Не так. Антиматерия живёт столько же, сколько материя, отличие - крошечное. Другое дело что в сверхранней Вселенной этого различия было достаточно.
Аноним 24/11/16 Чтв 12:57:19 #386 №397339 
>>397332
Хабропидора ответ.
Аноним 24/11/16 Чтв 13:04:14 #387 №397340 
>>397333
>Считается потому, что нигде не видно аннигиляции. Если-бы где-то в обозримом космосе были объекты из антиматерии - мы-бы видели, как они-бы аннигилировали.
Ну смотри одна галактика(и её мелкие спутники) полностью состоит из антиматерии, другая соседняя из материи, расстояния между ними может исчисляться дохуя световых лет(расстояние до ближайшей галактики Андромеды 2,5 миллиона световых лет), они очень врядли столкнуться и потому вряд ли ты увидишь охуенную аннигиляцию двух галактик
слабое взаимодействие и нарушение симметрии-кун
Аноним 24/11/16 Чтв 13:16:23 #388 №397342 
>>397340
Так-то оно да. Но не будет-ли видно аннигиляции всяких космических лучей?

Да и межгалактическое пространство - не идеально пустое.

Хотя не знаю, будет-ли заметна такая малоинтенсивная аннигиляция.
Аноним 24/11/16 Чтв 13:37:38 #389 №397343 
>>396328
Орууу жесть
мимоанон
Аноним 24/11/16 Чтв 13:41:13 #390 №397345 
Так классно, а я всего лишь про продолжительность ускорения корабля спросил с:
Аноним 24/11/16 Чтв 13:43:21 #391 №397346 
>>397345
В общем, подытоживая:

Для неподвижного наблюдателя ускорение будет постоянно падать, но никогда не станет строго равным 0.

Для команды корабля ускорение будет постоянным, но время Вселенной, что они будут в иллюминатор видеть, будет непрерывно ускоряться.
Аноним 24/11/16 Чтв 14:03:45 #392 №397349 
>>397346
/Сцы на острие научных теории.столетней давности
Аноним 24/11/16 Чтв 14:10:37 #393 №397350 
Посоны, я делал лабу по оптике и у меня период дифракционной решетки получился 0.02 мм, это вообще нормальное значение?
Аноним 24/11/16 Чтв 14:27:28 #394 №397353 
>>397285
>визуально ты не сможешь отличить воду от антиводы, хлор(зеленоватый газ) от антихлора(тоже будет зеленым), фтор(желтый газ) от антифтора(тоже будет желтым)
Пиздёж. Там как-то наёбываются сложные структуры из-за заряда другого.
Аноним 24/11/16 Чтв 14:30:01 #395 №397354 
>>397353
Нет. Материя и антиматерия симметричны, за исключением слабого взаимодействия.

https://ru.wikipedia.org/wiki/%D0%9D%D0%B0%D1%80%D1%83%D1%88%D0%B5%D0%BD%D0%B8%D0%B5_CP-%D0%B8%D0%BD%D0%B2%D0%B0%D1%80%D0%B8%D0%B0%D0%BD%D1%82%D0%BD%D0%BE%D1%81%D1%82%D0%B8
Аноним 24/11/16 Чтв 16:40:03 #396 №397366 
Напомните, а каким образом стало известно о существовании антиматерии?
Аноним 24/11/16 Чтв 18:34:27 #397 №397375 
>>397366
Это чисто гипотетическая сущность диалектического материализма. Ну, то есть жиды придумали.
Аноним 24/11/16 Чтв 19:20:06 #398 №397379 
>>397366
Дирак баловался с квантовыми уравнениями для электронов в атоме, в выводе оказалось существование антиматерии, через пару лет эксперементально нашли позитрон, а потом и остальные частицы, а потом и синтезировали антиводород и его аннигиляцию.
Аноним 24/11/16 Чтв 19:33:50 #399 №397381 
>антиматерия
Материя одна общая на всех, вот вещество может быть анти-не-анти. Не путайтесь позязя.
Аноним 24/11/16 Чтв 19:55:27 #400 №397383 
>>397381
ок, соре
Аноним 25/11/16 Птн 07:02:24 #401 №397409 
Нейтрон не участвует в электромагнитном взаимодействии или участвует, но имеет нулевой заряд?
Аноним 25/11/16 Птн 07:32:33 #402 №397410 
darksun.jpg
Если чёрная дыра просуществует достаточно долго, чтобы излучить предел энергии, необходимый для своего существования, она разуплотниться в нейтронную звезду?
Аноним 25/11/16 Птн 12:03:28 #403 №397421 
>>397375
при чем тут диамат вообще?
Аноним 25/11/16 Птн 12:37:12 #404 №397422 
>>397410
Ребята не стоит вскрывать эту тему. Вы молодые, шутливые, вам все легко. Это не то. Это не Чикатило и даже не архивы спецслужб. Сюда лучше не лезть. Серьезно, любой из вас будет жалеть. Лучше закройте тему и забудьте что тут писалось. Я вполне понимаю что данным сообщением вызову дополнительный интерес, но хочу сразу предостеречь пытливых - стоп. Остальные просто не найдут.
Аноним 25/11/16 Птн 16:47:19 #405 №397443 
>>397221
запас энергии фотона определяется так называемой длинной волны
если длинну волны считать отрицательной, то почему бы и нет?
Аноним 25/11/16 Птн 17:42:34 #406 №397448 
>>397410
Нет, у неё же в процессе испарения масса падает до планковской/нулевой (смотря какая теория верна). Если до планковской - то так и остаётся чёрная дыра планковской массы, если до нулевой - ничего не остаётся.
Аноним 26/11/16 Суб 02:11:51 #407 №397483 
Возможна ли телепортация? Вроде такие штуки называются кротовыми нормами.
Аноним 26/11/16 Суб 12:55:54 #408 №397495 
>>397483
Это смотря с какой стороны посмотреть.
Да. Специальная машинка измеряет каждую элементарную частицу твоего тела (в процессе ты разуплотняешься), передает инфу по проводам, удаленная машинка по этой инфе собирает твоего двойника из атомов палок, говна и опилок.
Нет. Нельзя единовременно узнать 100% инфы на квантовом уровне (принцип Гейзенберга), т.е. твой двойник будет на тебя только похож, но не идентичен.
> кротовыми нормами
Это не то, что бы телепортация.
Аноним 26/11/16 Суб 13:12:02 #409 №397496 
Как сделать скорость света отрицательной в практике?
Аноним 26/11/16 Суб 13:13:07 #410 №397497 
>>397495
Это будет мой двойник? Не я? Я уже не буду думать и жить?

другой
Аноним 26/11/16 Суб 13:58:46 #411 №397501 
>>397497
Парадокс тесейского корабля. При такой логике ты не рождался, а в пачпорте у тебя прописан человек, что уже не живёт, т.к. все атомы уже давно уплыли из организма. Человек это уже информационный организм, так что телепортация это тоже самое, что и загрузка в сеть и передача памяти на расстояние..
Аноним 26/11/16 Суб 14:00:00 #412 №397502 
Расскажите про эффекты, которые бы мы наблюдали при зернистости пространства.
sageАноним 26/11/16 Суб 14:55:59 #413 №397508 
>>397495
Только сейчас заметил, что очепятался.
Ты наверно понял, я хотел написать не про нормы, а про кротовые норы.
Аноним 26/11/16 Суб 14:57:04 #414 №397509 
>>397502
Оно зернистое.
мимо-гуманитарий
Аноним 26/11/16 Суб 16:13:47 #415 №397513 
>>396658
bump whoprosy
Аноним 26/11/16 Суб 16:56:54 #416 №397515 
>>393195
Если это штука, которая автоматически приходит в равновесие, то гугли тау, пид- lqr-регулятор и т.д. Также тебе понадобится Matlab для составления мат. модели твоего регулятора
Аноним 26/11/16 Суб 16:59:51 #417 №397516 
>>394211
То есть в черной дыре крутятся частитцы с отрицательной энергией?
Аноним 26/11/16 Суб 17:09:07 #418 №397517 
>>392830
Почему до сих пор нет таких аккумуляторов?
Аноним 26/11/16 Суб 17:44:54 #419 №397518 
>>393195
Такое на нечетком контроллере сделать ничего не стоит. Не нужны никакие рассчеты даже. http://ftp.funet.fi/pub/sci/neural/cns-bbs/annsim/togai.lzh
Аноним 26/11/16 Суб 17:57:02 #420 №397520 
>>397518
Только если ты хочешь сделать абы как, лишь бы работало. Потому что стоит чуть изменить массу и твоя нечеткая помойка уйдет в разнос.
Аноним 26/11/16 Суб 18:29:20 #421 №397521 
>>397501
Атомы уплывают по-одному, и ты в любой момент времени есть и неразрывен во времени, а при телепортации ты сначала умираешь нахуй, потом некоторое время (сто миллисекунд или сто тысяч лет) нет никого, потом машинка собирает искусственного человека по твоим примерным чертежам, прямо как трудолюбивый китаец собирает китайскую копию айфона.
Аноним 26/11/16 Суб 18:39:14 #422 №397524 
А разные атомы имеют разный радиус?
Аноним 26/11/16 Суб 19:30:18 #423 №397528 
>>397524
Конечно
Аноним 26/11/16 Суб 19:32:23 #424 №397529 
Что я увижу если стану фотоном?
Аноним 26/11/16 Суб 20:09:40 #425 №397532 
>>397524
Википедия учит нас:
"Согласно квантовой механике, атомы не имеют четких границ, но вероятность найти электрон, связанный с ядром данного атома, на определенном расстоянии от этого ядра быстро убывает с увеличением расстояния. Поэтому атому приписывают некоторый определённый радиус, полагая, что в шаре этого радиуса заключена подавляющая часть электронной плотности (порядка 90 процентов).
Характерной оценкой радиуса атома является 1 ангстрем (1 Å), равный 10-10 м."
Аноним 27/11/16 Вск 14:15:08 #426 №397561 
Мне тут могут помочь решить лабу? Это считается за тупой вопрос?
Аноним 27/11/16 Вск 22:55:09 #427 №397615 
>>397496
nu ребят
Аноним 27/11/16 Вск 22:55:31 #428 №397616 
>>397561
Если ты зажать вопрос
Аноним 27/11/16 Вск 23:05:20 #429 №397620 
>>397529
У тебя глаз не будет
Аноним 27/11/16 Вск 23:22:23 #430 №397627 
>>392513
ну если только твоей жеппы
Аноним 27/11/16 Вск 23:37:55 #431 №397630 
>>397620
А электроном по голове не ударит?
Аноним 27/11/16 Вск 23:47:32 #432 №397634 
>>395615
когда дойдут чипы до техпроцесса в 1 нм, тогда и будем говорить
Аноним 27/11/16 Вск 23:59:12 #433 №397636 
>>396613
точно платина
Аноним 28/11/16 Пнд 00:02:49 #434 №397637 
>>396690
2. нихуя не будет. главное шоб толщина была норм и сталь хорошая
Аноним 28/11/16 Пнд 00:05:22 #435 №397638 
>>396786
тогда уже кольцо, а не трубка
Аноним 28/11/16 Пнд 20:51:44 #436 №397709 
>>397140
пасиб
Аноним 30/11/16 Срд 23:26:12 #437 №397890 
Есть шанс, сделать камеру, система отчёта которой будет, в отношении Земли, двигаться со скоростью относительно близкой к скорости света. Дабы замедлить биологические процессы в ней. К примеру, использовать такую штуку по ночам, чтобы замедлить старение Путина/Катриарх/Кадырова? Но трабл в том, чтобы сама система приспособлений оставалась на месте. И это было, как в микроволновке, посредством электромагнитных волн разгонять частицы. Или я совсем дегрод?
Аноним 01/12/16 Чтв 00:05:16 #438 №397892 
>>397890
> или я совсем дегрод
Это.
Аноним 01/12/16 Чтв 02:30:16 #439 №397894 
>>397890
Да, есть. Только нужно еще изобрести квантовый расширитель пространства, что бы система отсчета двигалась с релятивистской скоростью, но навстречу ей с такой же скоростью все время набегало вновь созданное пространство из расширителя.
Аноним 01/12/16 Чтв 17:09:31 #440 №397928 
>>397890
хуйня в том, что биологические процессы в ней ты не замедлишь. если мимопутену будет суждено скопытится через 5 минут по его личному времени, он скопытится через 5 минут. просто на земле за эти 5 минут пройдет 5 веков например. иными словами, если твоя затея состоит в том, чтобы отправить какого нить присмерти опездола полетать, пока не изобретут лечение, то его один хуй оттуда потом достать например не успеют.
Аноним 03/12/16 Суб 20:08:53 #441 №398069 
>>390896 (OP)
1. Не остынет(Солнце будет греть), тепло будет передавать излучением.
2. Гравитация - это поле, а поле передаёт взаимодействие через элементарную частицу(гравитон). В этом году его впервые таки отловили, данные показывают, что скорость гравитационных волн скорее всего равна скорости электромагнитных. Исторически существовала теория дальнодействия, согласно ей, взаимодействие передаётся мгновенно и без посредников, но это опровергнуто.
Аноним 03/12/16 Суб 21:04:13 #442 №398074 
akv3.JPG
Так как фотоны умудряются отыскать кратчайший путь в воде? И если менять глубину, не трогая поверхность, то угол преломления будет меняться?
Аноним 03/12/16 Суб 22:39:27 #443 №398078 
>>398074
Попробуй это:
https://ru.wikipedia.org/wiki/Преломление
Аноним 04/12/16 Вск 00:41:10 #444 №398084 
>>398074
Фотоны ни какого оптимального пути сами не ищут. Просто вот ты светишь лазером, и свет преломляется так, что от твоей указки до точки на дне аквариума оптический путь минимален. Вот ты поменял глубину -- лазер светит уже в другую точку на дне, но время, которые летят фотоны от источника до этой точки опять минимально.
Аноним 04/12/16 Вск 00:47:00 #445 №398085 
>>397443
Ты совсем дегрод что ли? Ты любую волну можешь разложить в суперпозицию плоских. Для плоской волны отрицательная длина - это просто минус под синусом, это нихуя.
Аноним 04/12/16 Вск 03:00:23 #446 №398090 
>>398074
>кратчайший путь
>>398084
>оптический путь минимален
Кратчайший минимульный путь тут строго вниз. Что-то на пике нихуя фотоны его не находят.

>И если менять глубину, не трогая поверхность, то угол преломления будет меняться
Нет.
Аноним 04/12/16 Вск 11:03:21 #447 №398096 
>>398090
Ключевое слово "оптический".
Аноним 04/12/16 Вск 14:00:51 #448 №398111 
>>398096
Минимальный оптический путь - строго вниз. Если не согласен, иди гугли определение.
Аноним 04/12/16 Вск 16:11:12 #449 №398126 
>>398111
>Минимальный оптический путь - строго вниз
Минимальный путь до чего, блять? Ты либо зеленый, либо просто даун.
Аноним 04/12/16 Вск 16:21:32 #450 №398137 
>>398126
В глаза ебешься? Вот выше какой-то дебил ляпнул:
>>398084
>от твоей указки до точки на дне аквариума оптический путь минимален

А я ему и ответил, что на картинке нихуя не минимальный, минимальный будет строго вниз.
Аноним 04/12/16 Вск 16:45:49 #451 №398141 
>>398137
Ты деградант, путь от указки до ТОЧКИ, блять, НА ДНЕ. Не до дна, блять, и не до пизды твоей мамаши, а до точке, куда лазер, собственно, светит.
Нужна помощь!  Amiros 06/12/16 Втр 01:45:05 #452 №398214 
N7O7J24xul0.jpg
Помогите решить хотя-бы два задания.
Аноним 06/12/16 Втр 02:04:47 #453 №398219 
>>390896 (OP)
Посоны, если электрический ток это поток заряженных частиц, не будет ли работающий эл. чайник грубо говоря набирать массу, а провод из которого берутся электроны - худеть.
Аноним 06/12/16 Втр 02:14:29 #454 №398221 
Новый точечный рисунок.png
>>398141
Говно, какой путь минимальнее, h1 или h2?
Аноним 06/12/16 Втр 03:35:13 #455 №398222 
>>398219
Неа, ты представляешь поток заряженных частиц как бета-радиацию, но уместнее представлять ток, как поток жидкости в герметичной гидросистеме, нагнетаемый насосом (разностью потенциалов).
Аноним 06/12/16 Втр 14:49:50 #456 №398246 
14809796700260.png
>>398221
Аноним 06/12/16 Втр 14:51:11 #457 №398247 
>>398246
*Функционал
быстрофикс
Аноним 06/12/16 Втр 15:41:25 #458 №398250 
14810249905930.png
>>398246
Нашел путь поминимальнее.
Аноним 06/12/16 Втр 15:57:35 #459 №398254 
>>398250
Еще один, блять. Скорость света в воде и в воздухе разная. Представь, что ты стоишь на берегу озера, а в нем тонет твоя хуевая логика. Тебе нужно добраться до нее и спасти, но плаваешь ты плохо. Поэтому тебе быстрее пробежать чуть больше по берегу, чем бежать и плыть так, как ты нарисовал.
Аноним 06/12/16 Втр 16:04:31 #460 №398255 
>>398222
>нагнетаемый насосом (разностью потенциалов)
Тогда ты попадешь в ловушку хуевой аналогии, потому что в жидкости существует давление, и это есть результат взаимодействия молекул, а электроны никак особо друг друга не подгоняют, а ток в твоей сети существует за счет скин-эффекта.
Аноним 06/12/16 Втр 16:49:03 #461 №398261 
>>398255
Ты сам попал в ловушку бесполезного занудства. Суть аналогии была в том, что как и в герметичной гидросистеме в эл.цепи не будет места, где густо или пусто. Электрическое поле приводит в движение свободные электроны на всем протяжении цепи сразу, точнее, с задержкой обусловленной скоростью света, так же как насос приводит в движение всю жидкость с небольшой задержкой, обусловленной скоростью звука в жидкости.
Аноним 06/12/16 Втр 17:24:29 #462 №398264 
>>398261
Если человек рассуждает категориями водопровода, он постоянно делает смысловые ошибки. У меня есть опыт общения с радиотехниками/слесарями/прочими, их невозможно убедить в том, что они неправы, потому что они больше не могут абстрагироваться.
Аноним 06/12/16 Втр 17:26:11 #463 №398265 
>>398261
>Электрическое поле приводит в движение свободные электроны
А еще я уверен, что ты вообще не знаешь, о чем говоришь. И тебе кажется, что ты понимаешь, но на самом деле нет.
sageАноним 06/12/16 Втр 17:44:50 #464 №398269 
>>398264
>>398265
Ясно, ни слова конкретики от тебя, только кукаретика.
Аноним 06/12/16 Втр 17:48:59 #465 №398270 
>>398269
Попробуй перечитать еще раз.
Аноним 06/12/16 Втр 21:39:03 #466 №398284 
А вот, а вот элементарные частицы - это же квантовые объекты как ни крути? А вот если взять пузырьковую камеру или что-то такое, да? И насрать в нёё какими-нибудь ионизирующими уёбками вроде электронов, чью массу мы знаем. У нас есть треки => мы узнали координаты, так? Мы померили их длину - посчитали скорость, так? Мы теперь знаем и координаты, и скорость, выходит, что? Гейзенберг соснул? ЯННП
Аноним 06/12/16 Втр 22:18:07 #467 №398285 
>>398284
Ты забываешь про ширину трека, которая вполне микроскопическая. Вот она и неопределенность в положении электрона: ты знаешь только, что он внутри, грубо говоря, пузырька.
Аноним 06/12/16 Втр 22:26:01 #468 №398286 
>>398285
Но я могу же измерить ширину трека, так? Нано-штангенциркулем или еще чем, не важно. И получу я размер электрона, разве не так?
Аноним 06/12/16 Втр 22:45:52 #469 №398287 
>>398286
Размер пузырька -- что-то типа нескольких микрометров. Размер атома бора -- 1 ангстрем -- это в 10000 раз меньше. Классический радиус электрона -- еще в 100000 раз меньше.
Аноним 06/12/16 Втр 23:17:21 #470 №398288 
В вакууме, говорят, есть виртуальные частицы. Если космонафт в пустоте померяет например суммарный импульс кубометра вакуума и получит ноль, а потом начнет двигаться и получит не ноль, как это сочетается с тем, что не существует привелегированной системы отсчета?
Аноним 06/12/16 Втр 23:30:19 #471 №398289 
>>398288
Я мало что могу тебе рассказать про виртуальные частицы, но то, что ты написал - это какая-то глупость.
Аноним 06/12/16 Втр 23:42:05 #472 №398292 
>>398288
Вообще есть такая штука, как эффект Унру - появление теплового фона в ускоряющейся системе отсчета. Но Эйнштейн больше про инерциальные системы отсчета.
Аноним 07/12/16 Срд 01:27:00 #473 №398302 
>>398254
>ты стоишь на берегу озера, но плаваешь ты плохо
>>398264
>Если человек рассуждает категориями водопровода, он постоянно делает смысловые ошибки
Ну ты понел, да?

А вообще я рад, что мое сообщение пропало не зря и смогло зародить здравй смысл у уважаемых анонов. Не отмажешься теперь своими аналогиями. Кривая у него теперь короче прямой, лол.
Мимо, этот - >>398137
Аноним 07/12/16 Срд 18:13:02 #474 №398397 
>>398302
Просто ты ебанат, не знающий основ. Мне неприятно видеть, как такой, как ты, берет на себя ответственность объяснять вещи, которые не понимает.
Аноним 07/12/16 Срд 18:40:47 #475 №398401 
>>398397
>Мне неприятно
Зашивайся.
Аноним 07/12/16 Срд 20:04:10 #476 №398413 
Это правда, что вероятностный характер квантовой физики отражает её неполноту как теории? Значит ли это, что когда мы получим инструменты, способные оперировать с квантовыми объектами на квантовом уровне - например, микроскоп в который можно разглядеть фотон с длиной волны порядка типичного размера квантового объекта - то и во всех этих неопределенностях и плотностях вероятностей отпадёт нужда, теории микромира будут пересмотрены и мы придём к квантовым законам, не оперирующими понятиями плотностей вероятностей и фантазиями Гей-зенберга, а описывающие микромир подобно тому, как небесную механику описывают законы Кеплера?
Аноним 07/12/16 Срд 21:53:00 #477 №398416 
ioJio9DuXA0.jpg
>>398413
Прежде чем появится подобный микроскоп, искусственный интеллект (или прокачанный естественный) создаст математические модели с бесконечными рядами фрактальных производных, адекватно описывающие все квантовые процессы и создающие приборы по предсказанным на основании этого мат.представления новым эффектам.
Аноним 07/12/16 Срд 22:12:21 #478 №398417 
>>398413
Не означает, читай про неравенства Белла - если вкратце, то все эксперименты показывают, что у природы не может быть неизвестных нам параметров, которые оисывают не вероятностную картину. Но это не означает, что КМ полна
Аноним 07/12/16 Срд 23:22:34 #479 №398421 
>>398417
Я думал, теория Белла носит сугубо статистический характер. но да, количество зарегистрированных событий в экспериментах говорит нам о нарушении неравенств почти с полной уверенностью
Энивэй, вероятностный характери неполноту/неверность квантовой физики можно объяснить не только лишь одной теорией скрытых параметров, сам факт наличия абсурдных костылей вроде нелокальности или конфайнмента из КХД наталкивает на мысли. Придерживаюсь детерменизма. Бог не играет в кости. Луна не существует только тогда, когда я на неё смотрю.
Аноним 07/12/16 Срд 23:23:55 #480 №398422 
>>398413
>в который можно разглядеть фотон с длиной волны порядка типичного размера квантового объекта
Вот это хохма.

Вот ты хочешь разглядеть что-то очень маленькое. И чтобы иметь такую разрешающую способность, тебе нужна очень маленькая длина волны света, в котором ты будешь все это разглядывать. Проблема в том, что чем меньше длина волны, тем больше энергия кванта света и тем больше ты вмешиваешься в квантовую систему и меняешь ее параметры.
Аноним 07/12/16 Срд 23:28:22 #481 №398423 
>>398422
Я знаю. Если такая хохма тебе не заходит - можешь подумать об абстрактном штангенциркуле квантового размера. Или нейтринном микроскопе, чем-нибудь таком.
Аноним 07/12/16 Срд 23:35:38 #482 №398424 
>>398423
А еще я могу придумать абстрактное существо, которое будет нарушать второе начало термодинамики.

Штангенциркули такие существуют, кстати. Измеряют распределения плотности заряда. Но тоже вмешиваются в систему, к сожалению.
Аноним 08/12/16 Чтв 03:24:52 #483 №398430 
>>398413
>вероятностный характер квантовой физики отражает её неполноту как теории
Похоже что нет.
>инструменты, способные оперировать с квантовыми объектами на квантовом уровне
Эта хуета уже давно есть.
>микроскоп
Конечно, с этим примером пока напряг.

>>398424
Придумай. Потом появится моченый который скажет что не нарушает, а потом появятся другие моченые, которые запилят зверя в лабе.
http://m.phys.org/news/2016-02-physicists-photonic-maxwell-demon.html

Аноним 08/12/16 Чтв 11:47:32 #484 №398450 
>>398430
Ты в 2016 веришь в вечные двигатели, с тобой все понятно.
Аноним 08/12/16 Чтв 13:01:23 #485 №398451 
>>398450
При чем здесь вечные двигатели? Ты про демона хоть что-то кроме двух строчек лурки знаешь то? Хоть ссылки бы почитал, или тебе из них прямо отсыл в ПРЛ скопировать?
Аноним 08/12/16 Чтв 13:21:03 #486 №398453 
14791847665480.jpg
В книге Стругацких "За миллиард лет до конца света" главный герой Малянов вычисляет "М-полости" в планетарных туманностях.
Там псевдонаучный бред или что-то реально существующее описывается?
Аноним 08/12/16 Чтв 14:18:44 #487 №398461 
>>398451
Демон Максвелла -- это вечный двигатель второго рода, здравствуйте. Это значит, что он может превратить вообще все тепло в работу.
Аноним 08/12/16 Чтв 14:19:44 #488 №398462 
>>398397
Давай тебе станет еще неприятней: я с такими знаниями умудряюсь читать пары в одном из топ 400 вузов мира. По физике. Теперь подгорай.
Аноним 08/12/16 Чтв 14:23:59 #489 №398463 
>>398430
Каждый год какие-то Ахмеды придумывают нового демона Максвелла. И каждый раз оказывается, что они про что-то забыли.
Я чувствую почерк дауна, который не может в принцип Ферма. Почему ты еще не съебал?
Аноним 08/12/16 Чтв 14:32:17 #490 №398465 
>>398461
Так считали 50 лет назад. Я же говорю, "хоть что-то кроме двух строчек лурки"?

>>398463
Напиши комментарий в ПРЛ, тебя там все ждут с твоим авторитетным мнением.
Аноним 08/12/16 Чтв 14:33:41 #491 №398466 
>>398465
>50 лет назад
100 лет назад, быстрофикс.
Аноним 08/12/16 Чтв 14:37:54 #492 №398467 
>>398465
>Так считали 50 лет назад. Я же говорю, "хоть что-то кроме двух строчек лурки"?
Жду вразумительных аргументов, почему это не так.

>ПРЛ
Куда-куда?
Аноним 08/12/16 Чтв 14:48:03 #493 №398468 
>>398467
>Куда-куда?
В глаза ебешься или так на ссылку не нажал?
http://m.phys.org/news/2016-02-physicists-photonic-maxwell-demon.html
Там доступное объяснение, разжевывать не собираюсь.
Но я сегодня добрый, скопирую ссылку по сыылку напрямую. Статья то открытая:
http://journals.aps.org/prl/abstract/10.1103/PhysRevLett.116.050401
Аноним 08/12/16 Чтв 18:05:58 #494 №398482 
>>398468
Ты же понимаешь, что ты занимаешься словоблудием?

Ты предлагаешь мне придумать абстрактный прибор, который будет работать в разрез с принципом Гейзенберга.

При этом ты приводишь пример такого подхода: "Смотри, вот ученые же создали прибор, который должен опровергать второе начало термодинамики. И он его не опровергает."

Более того, дальше ты говоришь, что я дурачок, ведь уже 100 лет люди знают о том, что любое воплощение абстрактного прибора, который должен быть вечным двигателем, нихуя им не является.

Ты запутался в своей собственной пиздаболии, поздравляю.
Аноним 09/12/16 Птн 00:04:17 #495 №398496 
>>398453
Ну, один из Стругачей был астрономом, так что хз.
Аноним 09/12/16 Птн 00:16:06 #496 №398498 
zanyadfiz.jpg
Чиво вы тут спорите, я не понимаю. Это же квантовые частицы, у них своя атмосфера.
Аноним 09/12/16 Птн 01:53:54 #497 №398500 
>>398482
Уууух, какие манявры пошли.

>Ты предлагаешь мне придумать
Никто тебе ничего не предлагал, кроме как пойти нахуй. Ты сам ляпнул что
>А еще я могу придумать абстрактное существо, которое будет нарушать второе начало термодинамики.
На что тебе был дан ответ: уже давно известно что демон нихуя не нарушает.

>дальше ты говоришь, что я дурачок, ведь уже 100 лет люди знают о том
Нет, ты дурачок не потому что чего-то не знаешь, а потому что вместо того что бы потратить две минуты на ознакомление с темой - ссылка была в первом же посте - ты продолжал упорно жрать говно, а теперь маняврируешь горящим пуканом.

И да,
>можешь подумать об абстрактном штангенциркуле квантового размера
>Ты предлагаешь мне придумать абстрактный прибор, который будет работать в разрез с принципом Гейзенберга.
Давай ты тут быстро замолчишь, ведь я уже вижу что ты о Гойзенберге знаешь еще меньше чем о демоне.
Аноним 09/12/16 Птн 09:21:35 #498 №398509 
>>390914
>>390919
Про испарение металла забыли? Его вклад, конечно мал по сравнению с ИК, но существует.
Аноним 09/12/16 Птн 09:37:52 #499 №398510 
>>391233
Радиация может быть ионизирующим излучением, типа гамма и рентгена, может быть неионизирующим, например, УФ и видимый свет в ряде случаев. Есть еще заряженные частицы - альфа, бета и не только.

Для органики это плохо в основном потому, что органику разрушает - органические молекулы неспособны выдерживать такие энергии.

Радиация действует на клетки по-разному, в зависимости от типа. В подробностях могу пояснить только за повреждения ДНК, хотя и на остальные компоненты клетки тоже чувствительны.
1) Образование радикалов и разрушение ковалентных связей - при действии квантов с длиной волны меньше 280 нм, C-C связь в органике с очень высокой вероятностью разрывается, при этом образуются частицы с высокой реакционной способностью, биополимеры рвутся и иногда сшиваются не там, где нужно. В случае с ДНК это ведет к одно- и двуцеепочечным разрывам - такие повреждения крайнемутагенны.
2) Образование фотосшивок - при действии ультрафиолета на молекулу ДНК может образовываться тиминовый димер, два соседних основания соединяются напрямую ковалентной связью. Димер - довольно крупный аддукт и мешает процессингу ДНК (транскрипции и репликации) и его необходимо удалять. Если таких повреждений много - ферменты репарации перестают справляться и клетка умирает.

Но у радиации есть и положительные эффекты - например, возможность фотосинтеза, когда клетка поглощает кванты с определенной длиной волны и преобразует их в энергию.
Аноним 09/12/16 Птн 13:42:53 #500 №398525 
>>398500
>я уже вижу что ты о Гойзенберге знаешь еще меньше чем о демоне.

Ты снова проецируешь, мань. Нельзя компенсировать то, что ты обосрался, вымазыванием говном других.

>уже давно известно что демон нихуя не нарушает

Уже давно известно, что у тебя в одинаковой мере отсутствует как понимание физики, так и простая логика. Лишь такой даун, как ты, может путать абстрактный концепт с его воплощениями.

>Давай ты тут быстро замолчишь, ведь мне надоело оправдываться за то, что я слабоумный
Пофиксил
Аноним 09/12/16 Птн 14:34:12 #501 №398528 
Если на солнце произойдёт мощный взрыв мы сможем его услышать?
Аноним 09/12/16 Птн 14:41:43 #502 №398529 
>>398525
>путать абстрактный концепт с его воплощениями
То есть статью ты до сих пор так и не осилил, даже популярную. Про принцип Ландауэра ты само собой тоже не слышал. При этом продолжаешь срать и размазывать говно по стенам. Манявратор уровня 80.
Аноним 09/12/16 Птн 15:17:36 #503 №398534 
>>398529
>принцип Ландауэра

Как же ты, блять, заебал.

Можно придумать тысячи вариаций такого демона, который будет работать без знания информации о каждой молекуле. Попробуй, блять, сразу доказать, одним махом, что все они не будут работать. Для начала ознакомься с устройством ебучего храповика и собачки.

Просто иди нахуй, ебаный дегрод
Аноним 09/12/16 Птн 16:39:51 #504 №398540 
>>398528
Звуку передаваться не в чем. Да и далеко слишком.
Аноним 09/12/16 Птн 16:45:50 #505 №398541 
>>398528
На самом деле на солнце постоянно происходят термоядерные взрывы.
Аноним 09/12/16 Птн 16:48:26 #506 №398542 
>>398534
Странный ты, вместо того что бы почитать да разобраться, ты приходишь, съедаешь говна, бежишь читать на википедию и, нихуя не поняв, прибегаешь за говном снова. И так по кругу.
>такого демона, который будет работать без знания информации о каждой молекуле
А эта информация и не нужна, Ландауэр в даной задаче не туда и не так применяется. Но у тебя же даже и поп-пресс-релиз не получилось понять, что я тут стараюсь.

>>398509
>испарение металла
Ты хотел сказать сублподсказки нет, лол.
Аноним 09/12/16 Птн 16:58:03 #507 №398543 
Двачаны,кто нибудь хочет помочь мне с решением задач по электротехнике?
Аноним 09/12/16 Птн 17:01:03 #508 №398544 
>>392589
Кстати, а почему именно инфракрасный? Разве атомы не могут излучать энергию на меньшей частоте?
Аноним 09/12/16 Птн 17:01:30 #509 №398545 
>>398543
Нет.
Аноним 09/12/16 Птн 18:16:38 #510 №398548 
>>398500
Всю эту термодинамическую бредятину бредятину не я писал.
>>398413-кун
Аноним 09/12/16 Птн 18:23:08 #511 №398549 
>>398543
Сам ебись со своими комплексными хуйнями и Кирхгофами, студентомразь.
электрик-кун
Аноним 09/12/16 Птн 21:24:48 #512 №398556 
>>398548
Для тебя цитата из "Занимательной ядерной физики" здесь >>398498
Аноним 10/12/16 Суб 09:57:40 #513 №398571 
>>398540
>Звуку передаваться не в чем.
Из за вакуума или из за чего?
Аноним 10/12/16 Суб 13:06:35 #514 №398582 
>>398571
Нет, просто боженька запретил. Неча грязным людишкам слушать, что на небе происходит.
Аноним 11/12/16 Вск 09:44:43 #515 №398727 
>>390914
Пес, обьясни мне откуда тебе известно про притяжение центра галактики.? Ты кого наебушь, чмо? Это нереально установить.
Аноним 11/12/16 Вск 11:35:54 #516 №398733 
>>398727
Ты это кому? А центр галактики создаёт гравитационное поле и искривляет орбиты звёзд.
Аноним 11/12/16 Вск 14:49:09 #517 №398743 
>>398727
Это тебе нереально установить, ты же в глаза ебешься и не видишь кому вопрос задаешь.
А нормальным моченым такую элементарщину легко и просто установить и померять.
Аноним 12/12/16 Пнд 08:08:06 #518 №398802 
>>398542
Ок, сублимация, суть-то все равно уловил. Хотя происходить могут оба прцесса зависит от металла. А йод испаряется или сублимируется, умник?
Аноним 13/12/16 Втр 17:04:59 #519 №398918 
>>395676
>>395681
Лол, это что за хуйня, что за школьники такие, у меня интернета не было, я еще делить в столбик не умел, а уже знал что после вспышки молнии нужно щетать секунды до грома, а потом умножить их на 333, чтобы понять как далеко "ударила" молния, мб я и не понимал тогда как это работает в полной мере, но все таки знал, что это связано с ограничением скорости звука. Пиздос декаданс нравов и образования
Аноним 14/12/16 Срд 16:22:04 #520 №398994 
>>395676
>почему люди не видят задержку когда разговаривают с друг другом
Я вижу задержку. У тебя в развитии. Скорость звука сложно загуглить?
Аноним 14/12/16 Срд 18:38:37 #521 №399008 
>>395676
Ну хуй знает, я почему-то замечаю задержку звука при разговоре с людьми, когда специально обращаю на неё внимание.
Аноним 14/12/16 Срд 19:06:47 #522 №399013 
14736564014620.jpg
>>399008
К психиатру сходи.
Аноним 19/12/16 Пнд 05:58:58 #523 №399302 
14788611315670.png
поясните доходчиво за теорию времени пригожина и как она соотносится с симметриями в физике
Аноним 23/12/16 Птн 08:49:51 #524 №399675 
Сап, наукач.
Объясните мне, долбоёбу, что за хуйню несёт этот дед?
http://the-day-x.ru/russkij-uchenyj-vladimir-leonov-oprovergaet-xiggsa.html

Если я хоть немного понимаю нынешнее состояние физики, то дед порет хуйню в каждом абзаце.
Аноним 23/12/16 Птн 15:56:03 #525 №399698 
>>399675
Где конкретно хуйня? Вот с тем, что он не объясняет, почему не был обнаружен бозон Хиггса, я согласен.
Аноним 29/12/16 Чтв 01:01:18 #526 №400224 
При движения тела по мертвой петле частенько берут силу трения за ноль. Но тогда сила реакции опоры равна нулю, и тело, находясь в невесомосте в верхней точке петли, не движется по ней, не касается желоба (да, изначально тело движется по желобу и по-нему же идёт мертвой петлёй). Ну или просто коэффициент трения равен нулю.
Я это к тому, что тело, двигаясь по какой-то поверхности и совершая мертвую петлю, ни в одной точке движения не может быть в невесомости, ибо сила реакции опоры. Или што-то не так????????
Аноним 01/01/17 Вск 18:43:21 #527 №400578 
>>400224
В верхней точке тело движется параллельно поверхности, не касаясь его, так что сила реакции опоры равна нулю, а сила трения равна силе РО, умноженной на коэф. трения, то есть тоже ноль.
Аноним 06/01/17 Птн 00:29:39 #528 №401214 
Серия охуительных вопросов по черным дырам.

Если в черной дыре искажается пространство время, то любая материя как бы бесконечно падает в черную дыру никогда не образуя цельного вещества, поверхности и т.д.? Ну типа плотного тумана должно быть?

Как обстоят дела с версией о том, что черные дыры - это примерно тоже самое, что ядра атомов? Ну типа черная дыра - это как бы 1 атом, но огромный.

При падении в черную дыру объект будет разрушаться? Либо же изменение пространства времени приведет к тому, что космический корабль, например, будет лететь как ни в чем не бывало но в том пространстве, которое было затянуто за горизонт событий?

ну и как бы продолжая предыдущий вопрос - возможно ли такое, что внутри черной дыры не полный пиздец, гроб и кладбище, а просто кусочек вселенной, но плотно упакованный за счет искривление пространства - времени? Но то есть залетев туда ты окажешься в другой галактике или солнечной системе, целиком запакованной в этой хуйне.

И при такой раскладе - возможно и мы сами находимся уже внутри черной дыры?

Ну как-то так вот.
Аноним 06/01/17 Птн 03:15:10 #529 №401237 
>>401214
> При падении в черную дыру объект будет разрушаться?
У сверхмассивных черных дыр маленький градиент гравитации, поэтому не факт, что разрушится.
> возможно ли такое, что внутри черной дыры не полный пиздец, гроб и кладбище, а просто кусочек вселенной
Есть такая гипотеза, но распределение вещества внутри чд неизвестно, может там вакуум вокруг сингулярности, а в нем планетки.
> И при такой раскладе - возможно и мы сами находимся уже внутри черной дыры?
И такая гипотеза есть, но наверно, такая чд намного больше видимой части вселенной. Вообще, получается, для чд необязательно наличие центральной сингулярности.
Аноним 07/01/17 Суб 17:34:56 #530 №401437 
>>395627
>>395627
Ебать филологи собрались.
Кстати: Ебать, филологи собрались.
Аноним 10/01/17 Втр 21:03:26 #531 №401674 
>>401437
Оба варианта правильные.
Аноним 15/01/17 Вск 06:47:31 #532 №401980 
E=mc^2
Свет заходит в прозрачное стекло и замедляется.
Замедляясь свет теряет массу и энергию:
E1=m1c^2 m1<m E1<E
Проницаемость среды ухудшается, свет замедляется всё сильнее и в один прекрасный момент останавливается, а его
масса становится равной m0 массе покоя, а энергия фотона становится минимально возможной.
Выходит что измерив массу такой среды без света и с ним, мы могли бы найти массу света в покое.

Но свет же не может существовать в состоянии покоя.
А ещё мы нарушили закон сохранения энергии.

Свет до стекла имеет скорость с, в стекле скорость v<с, после стелка скорость c, итого свет спокойно меняет массу и энергию, нарушая при этом закон сохранения энергии.

Но решение найдено!
Свет отдаёт часть энергии в другое измерение Вселенной на хранение, а потом спокойно себе забирает эту энергию обратно!
Аноним 15/01/17 Вск 22:09:31 #533 №402039 
>>401980
Свет не имеет массы, долбоёб
Аноним 16/01/17 Пнд 02:00:34 #534 №402059 
>>402039
Полный унтерменш
Аноним 16/01/17 Пнд 04:50:15 #535 №402067 
>>402039
нет массы только у ничего, придурок хуесосный
Аноним 16/01/17 Пнд 05:58:17 #536 №402070 
>>402067
Свет это ничего.
Аноним 17/01/17 Втр 16:55:09 #537 №402162 
Сни11111мок.JPG
Сним1111ок.JPG
>>402067
>>402059
>>401980
Свет не имеет массы, но имеет импульс. Тем не менее энергия фотона зависит только от частоты, которая не меняется. Скорость световой волны в веществе уменьшается, но до нуля упасть не может, так как для этого нужен бесконечный показатель преломления. Массы покоя у света нет, так как нет самой массы. Если правда нет интересно, я приложил скрин с полной формулой. Если бы ты учил физику, тогда бы знал, что энергия всегда сохраняется и даже в самых крайних случаях не уходит ни в какие другие измерения. И пожалуйста, больше не выдумывай своих теорий, мамкин теоретик.
Аноним 17/01/17 Втр 16:56:32 #538 №402164 
>>402070
Свет - это волна частица.
Аноним 17/01/17 Втр 19:35:10 #539 №402183 
>>402162
У света есть масса в движении, и если его смогут заморозить, то у него тоже будет масса. А гравитация искривляет свет, и если бы у него не было массы, он выходил бы из чёрной дыры.

p=mv у света нет массы, но есть импульс? p=0v=0, что ты несёшь?

Как сохранятеся энергия, если по формулам она не сохраняется?
Аноним 18/01/17 Срд 05:06:47 #540 №402228 
>>401980
Замедляется скорость распространения, сами-то фотоны скачут между атомами со скоростью c.
Аноним 18/01/17 Срд 05:10:47 #541 №402229 
>>402183
Гравитация искривляет не свет, а пространство-время, следовательно, и траекторию световых волн.
И не p=mv, а p=h/λ.
> Как сохранятеся энергия, если по формулам она не сохраняется?
Так ты формулы сначала выучи, а потом сохраняй, лалка.
Аноним 18/01/17 Срд 08:59:20 #542 №402234 
>>402229
p=mv должно работать и для света

чёрная дыра в себя пространство-время засасывает? чего же тогда свет не выходит?
Аноним 18/01/17 Срд 09:10:23 #543 №402236 
p=mv=h/λ
m=p/v=h/(vλ)
v=c
m=h/(cλ)

Проблемы какие-то с массой света в движении? Тогда калькулятор в руки и вперёд.
Как вы поняли, масса света в движении зависит только от длины волны.
Аноним 18/01/17 Срд 09:15:33 #544 №402237 
>>402228
А в космосе между чем скачут фотоны?
Выходит, классическая скорость света "c" - которая именно скорость распространения света в вакууме, воовсе не константа?
Аноним 18/01/17 Срд 09:28:24 #545 №402238 
>>402236
добавка
E=hню
E=mc^2
m=h
ню/с^2
Аноним 18/01/17 Срд 10:01:34 #546 №402241 
>>402183
Вот жеж дебил, так трудно осилить 1 страницу из википедии, которую я оставил. И не надо спорить, если тебе говорят, что земля круглая.
Аноним 18/01/17 Срд 10:02:34 #547 №402242 
Ребят, может хватит применять классическую механику к свету?
Аноним 18/01/17 Срд 10:02:46 #548 №402243 
htr7bx.jpg
>>402238
Продолжаем мысль:

А эта формула нам показывает, что мы по массе-распостранения в движении, можем найти и массу-распространения покоя по формуле:
m0=m (выходит, только экспериментально, постепенно приближаясь к v=0)
m0 тоже зависит от длины волны и поэтому длина волны при прохождении света через среду увеличивается:
m=h/(vλ) v - скорость света в среде

А вот с массами-меж-квантовыми-переходами возникает небольшая проблема - ну между атомами.
Аноним 18/01/17 Срд 10:04:15 #549 №402244 
>>402241
так мне же, полный дебил доказывает, что Земля стоит на трёх китах и это же в Википедии написал чуть меньший дебил.
Аноним 18/01/17 Срд 10:05:46 #550 №402245 
>>402244
Ну давай, ещё с википедией поспорь. Ты же у нас самый умный. Там между прочим есть ссылки на первоисточники и литературу.
Аноним 18/01/17 Срд 10:07:00 #551 №402246 
>>402244
И то, что земля стоит на черепахе, пытаешься доказать ты.
Аноним 18/01/17 Срд 10:09:29 #552 №402247 
>>402245
ну так я и первоисточники читал
Аноним 18/01/17 Срд 10:10:23 #553 №402248 
>>402247
Это какие, физика за 7 класс?
Аноним 18/01/17 Срд 10:12:23 #554 №402249 
>>402248
с тупым школьником, имхо, говорить не о чем, подрастай пока
Аноним 18/01/17 Срд 10:13:42 #555 №402250 
>>402249
Домашку сделать не забыл? По физике особенно
Аноним 18/01/17 Срд 10:25:09 #556 №402251 
>>402243
Эта формула используется для тел с массой. А если считать по ней массу света, получится, что в вакууме в знаменателе ноль 1-c^2/c^2=0 и при массе покоя, не равной нулю, получится, что масса света равна бесконечности.
Аноним 18/01/17 Срд 10:26:33 #557 №402252 
14843210367900.jpg
>>402243

Продолжаем мысль:
А при достижени m0, у света будет максимально большая длина волны равная... БЕСКОНЕЧНОСТИ!!!
m0=h/(0λ)

Кроме того, по опыту могу сказать, что в данном случае 0λ - ноль умноженный на бесконечность должны дать один, а не ноль,
и m0=h/(0λ)=h/1=h!!!

Именно!
m0=h !!!

Масса покоя электромагнитной волны численно равна Постоянной Планка!!! ДА!!!
Аноним 18/01/17 Срд 10:28:08 #558 №402253 
>>402252
это такой троллинг?
Аноним 18/01/17 Срд 10:38:18 #559 №402256 
14843210367900.jpg
>>402252

Итого, что выходит, производная примерна, Минковский, Лоренц и Эйнштейн ошиблись (m0=h/(0λ) получилась не совсем понятной, но если принципы сохранились...), а в наших формулах их ошибки и приближения сократились, после чего из неправильных и приближённых формул и законов мы получили правильный вывод! (что похоже на полный идиотизм, либо намеренно подстроенное событие)

m0=h
Где m0 - масса покоя электромагнитной волны, h - постоянна планка.
Аноним 18/01/17 Срд 10:45:45 #560 №402257 
ris43.gif
>>402252
Масса покоя света равна нулю. Это факт. Ноль, умноженный на бесконечность даёт неопределённость. Формула зависимости релятивистской массы от массы покоя применяется только для тел, имеющих массу. Безмассовые частицы принципиально отличаются от частиц с массой. Из формулы зависимости массы от скорости видно, что у тел с массой при увеличении скорости возрастает энергия и она может расти бесконечно, так как нельзя разогнать тело с массой до скорости света (так как инерция тела возрастает и сила придаёт ему всё меньшее ускорение), а свет же наоборот всегда передвигается с максимальной скоростью в среде, а его энергия постоянна.
Аноним 18/01/17 Срд 10:55:06 #561 №402259 
Снимsaaasasasок.JPG
>>402249
12 ссылок, если бы ты не был деградантом, для тебя бы это что-то значило.
Аноним 18/01/17 Срд 11:00:20 #562 №402260 
>>402257
Ноль на бесконечность конкретно в данном случае даёт один.
Не имеет массы только ничто.
Не нужно мне википедию перечитывать, её пишут конченные неудачники, которые даже до конца не понимают то, что они пишут, это ведь не их открытия, они только из книжек инфу переписывают, а книжки я читал.
Про формулы говорить не надо, они не верны.
Когда я включая фонарь, я не трачу всю энергию Вселенной для этого, хватит подобных глупостей - заблуждения из-за неверных формул. Не имеет массы только ничто.
Аноним 18/01/17 Срд 11:01:51 #563 №402262 
>>402260
Окей, ты умнее всего мира. Удачи, Энштейн.
Аноним 18/01/17 Срд 11:07:10 #564 №402264 
>>402257
Да, вас просто бесит, что формулу и закон мироздания открыл я, а не вы.

m0=h
Где m0 - масса покоя электромагнитной волны, h - постоянна Планка.
Аноним 18/01/17 Срд 11:08:29 #565 №402265 
>>402264
Слушай, займись чем-нибудь полезным. Маме хотя бы по дому помоги.
Аноним 18/01/17 Срд 11:45:11 #566 №402269 
hypothesis1.png
Сделал картинку, можете ставить эксперименты.
Аноним 18/01/17 Срд 11:52:25 #567 №402270 
>>402269
Даже не ебёт, что постоянная планка измеряется в Джулях на секунду.
Аноним 18/01/17 Срд 12:18:16 #568 №402271 
>>402270
m0=h/(vλ)=h/1=h

кг=(кг м^2/c)/(м/с м)=(кг м^2/c) x с/м^2
m0=h x 1с/м^2
Аноним 18/01/17 Срд 12:47:00 #569 №402276 
hypothesis1.png
Дополнил, чтобы ни у кого больше не было никаких вопросов.
Аноним 18/01/17 Срд 13:35:11 #570 №402279 
>>402276
Поздравляю, ты взорвал мой пукан. А теперь иди, учи уроки.
Аноним 18/01/17 Срд 14:45:51 #571 №402291 DELETED
>>402237
Между твоих ягодиц.
Аноним 18/01/17 Срд 15:38:07 #572 №402293 DELETED
>>402291
говорят, латентный гомосексуализм излечим, сходи к доктору
Аноним 18/01/17 Срд 15:57:08 #573 №402294 
>>402276
Давайте доказательство тогда сделаем более понятное (мы говорим про распространение света):
ню - это одна буква - это частота
p=mc=hню/c
Скорость падает
p=mv=hnню/v
m=hnню Движение всё ещё есть.
Итак, масса в данном случае не зависит от скорости, а зависит от частоты и показателя преломления.

Теперь происходит полная остановка.
Учитываем погрешности и неправильность формул, хуяк, хуяк, n и ню - бесконечность с нулём чпокаются и мы, как пить дать, получаем:
m0=h

Осталось проверить это экспериментально и если подтвердится, подправить СТО, ОТО и квантовую физику.
Аноним 18/01/17 Срд 16:09:17 #574 №402295 
>>402294
пиздец, сука, а я то думал чего не сходится!
за целый день расчётов устал и начал делать тупые ошибки.
куда анон смотрит, не понятно
Аноним 18/01/17 Срд 16:26:20 #575 №402299 
fixed
Давайте доказательство тогда сделаем более понятное (мы говорим про распространение света):
ню - это одна буква - это частота
n - показатель приломления
p=mc=hню/c

Скорость света в среде падает:
p=mv=hnню/v
m=hnню/v^2
v/λ=nню m=(hv/λ)/v^2=h/λv Возможно, всё примерно.
m=h/λv Это формула из первого описания гипотезы.

До сих пор движение сохранялось.

Теперь происходит полная остановка.
Учитываем погрешности и неправильность формул, хуяк, хуяк, n, ню и v; λ и v - бесконечность с нулём чпокаются и мы, как пить дать, получаем:
m0=h

Осталось проверить это экспериментально и если подтвердится, подправить СТО, ОТО и квантовую физику.
Аноним 18/01/17 Срд 18:23:01 #576 №402308 
есть ошибка, исправлю завтра
Аноним 18/01/17 Срд 19:18:23 #577 №402312 
14843210367900.jpg
>>402299
Длина волны (λ) в среде должна быть короче, чем в вакууме:
λ=с/f f - частота
λ=v/f=c/nf фазовая скорость v<c
у нас же наборот:
m=h/(vλ) Раз m и v уменьшаются, а h - константа, то λ должен возрастать.

вакуум:
p=mc=hf/c=h/λ
m=p/c=hf/(c^2)=h/cλ
среда:
p=mv=hf/v=hnf/c=h/λ
m=p/v=hf/(v^2)=hnf/(vc)=h/(vλ)
m=h/(vλ)
непроходимая среда (предположительно):
m0=h

Вот теперь вроде всё правильно по формулам.
А доказать предстоит, что при v->0, vλ -> 1.
Не будем забывать, что это гипотеза, и она может оказаться не верна скорее, чем верна (статистика).
Аноним 18/01/17 Срд 22:25:35 #578 №402328 
>>402312
>вроде всё правильно
Если не считать ошибки, про которые я сказал. Ты ведь всё равно не слушаешь, что тебе говорят. Ну да ладно, я всё равно объясню. при умножении бесконечности на ноль получается неопределённость
∞0=x
x/0=∞ x/∞=0
здесь x может принимать любые значения, включая бесконечность
Формула для нахождения релятивистской массы из массы покоя применяется только для тел с массой. Чёрным по белому написано. Так же, как и написано то, что у света нет массы покоя. И бесконечного показателя преломления добиться невозможно. Так же, как нельзя разогнать тело с массой до скорости света, нельзя замедлить скорость света до нуля.
Ещё меня смущает, что ты находишь массу из импульса по классической формуле. Имхо, это должно делаться по-другому.
Аноним 19/01/17 Чтв 05:04:39 #579 №402350 
14843210367900.jpg
>>402312
Второе блиближение нашей формулы (на свежую голову).

1) m0=h - это в самом лучшем случае.
2) m0=k1h - где k1 - постоянный коэффициент (вроде pi, h итд)
2) m0=k2h - где k2 - динамический коэффициент (вроде n), который находится по некоторой формуле.

Проводим массу экспериментов с разнымы частотами и длинами, пытаемся остановить свет как можно сильнее и сделать n как можно больше, находим закономерности в результатах и составляем нашу формулу.

Наша задача: подогнать m0 электомагнитной волны к планковским масштабам, т.е. m0 мы подгоняем к h любыми путями (m0=xh), но чтобы числа, которые мы получили в результате экспериментов сходились.

После и во время чего нужно будет посмотреть
на формулы импульсов, которые мы взяли и подправить их так, чтобы при приравнивании получалась наша формула, там явно что-то не так, мне она не нравится.
Аноним 19/01/17 Чтв 05:07:16 #580 №402351 
>>402350
Ещё нужно будет внимательно посмотреть что из себя представляет kh.
Аноним 19/01/17 Чтв 05:09:04 #581 №402352 
>>402350
k1 может быть постоянным коэф, который находится по некоторой формуле.
Аноним 19/01/17 Чтв 05:59:03 #582 №402353 
>>402328
я знаю что такое ноль, бесконечность и неопределёность.

а блаблаблаблабла

если ты про СТО, так она ошибочна

я могу замедлить скорость света до нуля: две стенки сдвигаются друг к другу, свет идёт меджду ними, чпок, в один прекрасный момент не идёт и во Вселенной он от одного конца до другого не доходит, проблемы какие-то? Ах да, нам же нужно постепенное и наглядное v->0, ну тогда берём среду с изменяющимся n, пускаем свет и в один прекрасный момент оказывается, что свет до куда-то не дошёл, а когда мы начнём смотреть перед той точкой мы и v почти ноль найдём, а может и точку v=0.

Вот ты лично откуда знаешь, что у света нет массы покоя? Мы пытаемся доказать обратное, и, возможно, нам это удастся.
Я включаю фонарь и из него вылетает свет с массой в движении.
Не надо больше про СТО.

Должно делаться иначе? - Ваши предложения.
Аноним 19/01/17 Чтв 06:24:01 #583 №402354 
>>402350
* второе приближение, бля
Аноним 19/01/17 Чтв 14:48:43 #584 №402380 
>>402353
>Должно делаться иначе? - Ваши предложения.
Например сделать всё по научному
Сделай научную статью, в которой с аргументами последовательно опровергаешь СТО и опубликуй её.
Аноним 19/01/17 Чтв 17:44:28 #585 №402400 
u56j.png
>>402380
оно мне надо?
Аноним 20/01/17 Птн 10:46:59 #586 №402444 
>>402400
Ну тогда не кукарекай
Аноним 20/01/17 Птн 18:52:14 #587 №402472 
>>402444
базар фильтруй, опущ
Аноним 28/02/17 Втр 15:23:56 #588 №405640 
>>393195

https://www.youtube.com/watch?v=-7xvqQeoA8c
Аноним 28/02/17 Втр 15:24:48 #589 №405641 
>>405640 это - скорее сюда -> >>393303
comments powered by Disqus

Отзывы и предложения